You are on page 1of 56

1

II Bienal da SBM
Salvador/BA - 25 a 29 de Outubro de 2004

Olimp ada de Matem atica - Uma porta para o futuro

Dicas para montar um projeto e 50 problemas de treinamento para iniciantes

Emanuel Carneiro

Apresenta c ao
Os livros nos contam que, em tempos mais antigos, matem aticos desaavam uns aos outros propondo quest oes complicadas e por muitas vezes se reuniam em pra ca p ublica para a realiza ca o de torneios, onde teriam que resolver equa co es dif ceis. O que nasceu talvez por um capricho do ego destas pessoas tomou forma mais salutar com a realiza ca o da 1a Olimp ada de Matem atica, na Hungria em 1896. De l a pra c a, as competi co es de matem atica entre estudantes v em a cada dia se organizando e se mostrando um forte indicador para descobrir novos talentos para a ci encia. Em 1959, foi realizada a 1a Olimp ada Internacional de Matem atica (IMO), com ber co no leste europeu e contando apenas com a presen ca de pa ses comunistas. Em 1965 a Finl andia se torna o primeiro pa s n ao comunista a fazer parte do hall de participantes da IMO, abrindo o caminho para que esta competi ca o viesse a ser a maior de matem atica do mundo atual (hoje na sua edi ca o 46), com cerca de 85 pa ses participantes por ano e que re une em m edia 500 estudantes de todo o planeta. Esse evento mundial estimulou a cria ca o de p olos em cada regi ao. Assim, hoje em dia, cada pa s conta com sua Olimp ada Nacional de Matem atica, e por um racioc nio an alogo, as prov ncias destes pa ses tamb em resolveram organizar competi co es de matem atica e at e os pr oprios munic pios. Foi assim que tudo come cou em nosso pa s. Em 1979 foi realizada a 1a Olimp ada Brasileira de Matem atica, por iniciativa amadora de alguns matem aticos. A OBM est a hoje em sua 26a edi ca o, abrangendo cada vez lugares mais remotos em nosso pa s, contando com a participa ca o anual de cerca de 150.000 alunos. V arios estados brasileiros tamb em j a contam hoje com sua pr opria Olimp ada de Matem atica, numa prova de evidente crescimento do interesse das pessoas por estas competi co es. Por muito tempo, o trabalho de se organizar as competi co es de matem atica em nosso pa s foi completamente amador, feito por admiradores incondicionais da matem atica e de seus segredos. Apenas h a alguns anos (em 1998), uma verba do governo foi destinada para ns de apoio, e com isso a OBM cresceu em estrutura e em n umero de adeptos. O resultado do Brasil na IMO vem se consolidando a cada ano, hoje ocupando posi ca o de destaque entre os 20 melhores pa ses do mundo. Sim, ao contr ario do que muitas pessoas pensam, o Brasil e bom em matem atica, sim! Por enquanto talvez s o na matem atica de ponta, mas o mais importante e que isso foi conquistado em relativamente pouco tempo, o que nos impulsiona a acreditar mais e mais no POTENCIAL que o brasileiro tem para aprender. Este texto e sobre isso. Sobre o POTENCIAL de nossos estudantes que cam em sua maioria encurralados por um ensino p ublico decadente. Sobre a entidade Olimp ada de Matem atica que cresce em nosso pa s, como instrumento de desenvolvimento da educa ca o e processo de inclus ao social indiscut vel pois oferece muitas oportunidades a jovens independentemente de ra ca, sexo e lugar onde mora. Gostaria de apresentar a voc es minha vis ao das coisas e mostrar o que se pode fazer com um pouquinho de dedicac a o e vontade de melhorar. Quero apresentar a Olimp ada de Matem atica n ao como um m, mas como um dos meios, uma op ca o, para reerguermos o ensino de matem atica no Brasil. Da o t tulo do texto.

Emanuel Carneiro - Fortaleza - Setembro de 2004

Cap tulo 1

O que a matem atica de Olimp ada tem de diferente: alguns exemplos


At e aqui, j a falamos um pouco sobre a Olimp ada de Matem atica, o hist orico e as competi co es existentes no Brasil e no mundo. Chegou ent ao a hora de conhecer um pouco dos problemas que costumam compor as provas destas competi co es. Erroneamente, muitas pessoas pensam que estudar para participar de uma Olimp ada de Matem atica e avan car na mat eria usual do col egio, que por exemplo um aluno da 5a s erie deva estudar equa ca o do 2o grau, ou um aluno do ensino m edio deva saber tudo de c alculo. N ao e nada disso. Os problemas n ao exigem uma dose maior de conhecimento, e sim o despertar de um racioc nio e de muita criatividade. O aluno e for cado a experimentar sua intelig encia, diferentemente da maioria dos problemas mec anicos que fazem parte do curr culo escolar. Como primeiro exemplo, o seguinte problema foi proposto na Olimp ada de Matem atica da Escola P ublica do Estado do Cear a em 2003, na modalidade II (1o ano do ensino m edio): Exemplo 1.0.1 S ao dadas 4 moedas aparentemente iguais. Sabe-se que uma delas e falsa (tem peso diferente das demais). Mostre como descobrir a moeda falsa com 2 pesagens em uma balan ca de dois pratos. Tentem pensar alguns minutos antes de olhar a solu ca o... Solu c ao: Chame as moedas de A, B, C, D. Pese duas delas, digamos A e B . Temos ent ao duas possibilidades: H a equil brio entre A e B : Nesse caso A e B devem ser moedas leg timas. Pese ent ao A com C . Se equilibrar de novo, a falsa dever a ser a moeda D. Se n ao equilibrar, C ser a a falsa. N ao h a equil brio entre A e B : Conclu mos da que A ou B e a falsa. Pese ent ao A e C (que e uma moeda verdadeira). Se houver equil brio, B ser a a falsa. Se desequilibrar, A ser a a falsa. De qualquer modo voc e consegue ent ao descobrir qual a moeda falsa usando duas pesagens. Coment arios: Ap os ter visto a solu ca o deste problema, podemos destacar alguns pontos: Apesar de ter sido proposto para a modalidade II (1o ano do ensino m edio), este problema poderia ser resolvido por alunos do ensino fundamental, j a que s ao necess arios apenas racioc nio e um pouco de paci encia. A maioria dos alunos ao olhar um problema deste tipo pensa 1 minuto e diz que n ao sabe fazer. Isso ocorre porque eles est ao acostumados com uma matem atica r apida, de muita conta e pouca id eia. Se o aluno for devidamente estimulado (ofere ca um pr emio!) certamente ele vai fazer. Como foi visto na solu ca o, o aluno ao fazer uma prova de Olimp ada de Matem atica, deve tamb em ter um bom dom nio do portugu es, j a que o u nico modo que ele tem para expor suas id eias e atrav es do l apis e do papel. Muitas vezes alunos saem das provas dizendo: - Ah professor, essa quest ao eu sabia fazer, s o n ao consegui explicar direito...

N ao adiantou nada. A pessoa que vai corrigir vai pensar que ele n ao sabia. Resumindo, est a em jogo aqui n ao s o a capacidade de racioc nio do aluno, mas tamb em sua capacidade de comunica ca o escrita e a sua organiza ca o (anal, s o n os professores sabemos como e chato corrigir provas cheias de borr oes innd aveis...) Ap os este primeiro contato, podemos ent ao ver um segundo exemplo. Exemplo 1.0.2 (i) Mostre como dispor os n umeros 1, 2, 3, 4, 5, 6, 7, 8, 9 nas casas de um quadrado 3 3 para formar um quadrado m agico, ou seja, para que a soma dos n umeros nas linhas, colunas e diagonais seja a mesma. (ii) Prove que em todo quadrado m agico o n umero da casa central deve ser o 5. Novamente, pense um pouco antes de ver a solu ca o. Solu c ao: (i) H a v arias maneiras. Por exemplo: 4 9 2 3 5 7 8 1 6 6 7 2 1 5 9 8 3 4

(ii) Digamos agora que a soma comum (das linhas, colunas e diagonais) seja S . Vejamos ent ao uma distribui ca o qualquer para um quadrado m agico: a d g Somando as tr es express oes temos: a + b + c + d + e + f + g + h + i = 3S 1 + 2 + 3 + 4 + 5 + 6 + 7 + 8 + 9 = 45 = 3S S = 15 Por enquanto, s o usamos as somas dos n umeros de cada linha. Temos ainda uma por ca o de somas que podemos utilizar. Estrategicamente escolhemos as seguintes: a + e + i = 15 g + e + c = 15 d + e + f = 15 b + e + h = 15 Somando estas u ltimas quatro express oes, obteremos: (a + e + i) + (g + e + c) + (d + e + f ) + (b + e + h) = 60 (a + b + c + d + e + +f + g + h + i) + 3e = 60 (1 + 2 + 3 + ... + 9) + 3e = 60 45 + 3e = 60 e=5 b e h c f i a+b+c d+e+f g+h+i = S = S = S

M agica?

O que voc es acharam ent ao deste segundo exemplo? O primeiro item e um otimo exerc cio para qualquer idade. Imagina a felicidade de um aluno de 10 ou 11 anos que consiga resolv e-lo...J a o segundo item e um pouco mais elaborado. O aluno pode ent ao perguntar: -Professor, me diga como e que eu conseguiria fazer isso? Eu jamais teria essas id eias.

- Tudo bem, voc e n ao sabia. Agora eu j a mostrei pra voc e. Da pr oxima vez ent ao que aparecer algo assim, voc e j a pode come car a ter estas id eias... A melhor forma de uma pessoa aprender matem atica e resolvendo exerc cios. Quando n ao sabemos resolver algo, isso n ao deve ser o m do mundo, anal ningu em e obrigado a saber tudo. Neste ponto devemos chegar para algu em mais experiente para que esta pessoa nos d e novas id eias, para que ela nos passe o bizu e nos ensine a resolver o maldito problema. Deste momento em diante, teremos estas novas id eias em nosso arsenal e quando aparecer outro problema parecido... O pr oximo exemplo e um pouco mais elaborado, mas n ao menos interessante: Exemplo 1.0.3 Mostre que n ao h a inteiros mpares a, b, c, d, e, f tais que: 1 1 1 1 1 1 + + + + + =1 a b c d e f Solu c ao: Se isto fosse poss vel, ter amos: bcdef + acdef + abdef + abcef + abcdf + abcde =1 abcdef bcdef + acdef + abdef + abcef + abcdf + abcde = abcdef

Nesta u ltima express ao temos do lado esquerdo a soma de seis n umeros mpares, que sabemos que e par. Por em, do lado direito temos um n umero mpar (abcdef ). Absurdo!! Coment arios: A solu ca o acima j a e um tanto quanto mais sosticada. N ao que seja dif cil, pois acredito que a maioria das pessoas est a apta a compreend e-la. Por em, a solu ca o usa duas ferramentas b asicas em matem atica: o m etodo de redu ca o ao absurdo e os argumentos de paridade (um n umero ou e par ou mpar, jamais os dois ao mesmo tempo). Se o aluno n ao conhece estas ferramentas, ele vai certamente encontrar diculdade. Novamente voltamos ` a conversa do exemplo anterior. Cabe ent ao ao professor ensinar as armas b asicas (e nestas o absurdo e a paridade est ao certamente inclu das) para que seus pupilos possam achar estas id eias naturais. Um fato curioso e que a maioria dos alunos conhece a paridade dos n umeros (anal eles devem jogar par-ou- mpar para sobreviver), mas cam extremamente abismados quando v eem que argumentos de paridade possam ser usados para resolver muitos problemas de matem atica. No pr oximo exemplo vemos um t opico que est a entre os mais queridos na matem atica, e que aparece com freq u encia nas competi co es: geometria. Exemplo 1.0.4 No tri angulo ABC abaixo, BP e bissetriz do a ngulo B e M e o ponto m edio do lado AC. Se AB = 6 e BC = 10, calcule PM. B

6 P A M

10

Solu c ao: Veja a nova gura, onde prolongamos AP at e encontrar o lado BC em Z.

6 P A M

10

Note que no tri angulo ABZ, o segmento AP e altura e bissetriz. Isso faz com que o tri angulo ABZ seja is osceles! Logo BZ = AB = 6 e portanto: ZC = BC BZ = 10 6 = 4 Perceba ainda que como o tri angulo ABZ e is osceles, BP e altura, bissetriz,... e mediana! Logo P e o ponto m edio de AZ. Como M j a e o ponto m edio de AC, vemos que PM e a base m edia no tri angulo AZC. Conclus ao: PM = ZC =2 2

E este agora, o que acharam? A geometria plana e um dos t opicos que mais cativam os estudantes. Acho que eles se sentem felizes fazendo desenhos. Isso e bom, pois na pr atica 30% dos problemas propostos em olimp adas de matem atica s ao de geometria. Agora vamos a alguns exemplos voltados para alunos do ensino m edio. Anteriormente vimos problemas de matem atica que envolviam muito racioc nio e criatividade, mas nem sempre isso pode ser suciente. Se um aluno deseja participar da Olimp ada de Matem atica e obter sucesso, uma condi ca o necess aria e que ele tenha o dom nio da mat eria do col egio, dos fundamentos de matem atica. Que ele tenha certa base. Isso e mais not avel quando mostramos os problemas ol mpicos para o ensino m edio, pois nesses o aluno deve aliar sua intelig encia ` a sua maturidade matem atica. O pr oximo exemplo e muito bacana. Foi quest ao da Olimp ada Brasileira de Matem atica, 1a fase, em 2002. Exemplo 1.0.5 Seja a maior raiz da equa ca o x2 x 1 = 0. Calcule 5 5. Este realmente vale 5 minutos do seu tempo... Solu c ao: Usando o fato que 2 1 = 0, temos: 2 4 Assim conclu mos que: 5 5 = 3 Coment arios: Muitas vezes as pessoas esquecem de usar a pr opria deni ca o de raiz de uma equa ca o. Que este e um n umero que posto no lugar do x vai zerar a equa ca o. Foi isso que zemos na solu ca o. A maioria dos estudantes, quando escuta a palavra raiz, vai longo chamando a f ormula de Bh askara para encontrar: 1+ 5 = 2 e vai fazendo um mont ao de contas...
3

+1

= ( + 1) = 2 + = 2 + 1 = (2 + 1) = 22 + = 3 + 2 = (3 + 2) = 32 + 2 = 5 + 3

Isso n ao impede que ele resolva o problema. A entra o que dissemos anteriormente. Se ele for um bom aluno do ensino m edio e tiver aprendido bin omio de Newton ele vai conseguir calcular resposta. Vai dar s o um pouquinho mais de trabalho...
1+ 5 2 5

, e vai encontrar a

Para encerrar ent ao esta pequena amostra de problemas interessantes em matem atica, a vai um bem legal: Exemplo 1.0.6 Prove que a fun ca o f : R R denida por:

f (x) = cos x + cos(x 2)

n ao e peri odica. Solu c ao: Vamos novamente usar o m etodo de redu ca o ao absurdo (isso realmente e importante!). Suponha que nossa fun ca o f seja peri odica de per odo p. Isso quer dizer que: f (p + x) = f (x) para qualquer x R. Em particular, tomando x = 0, temos:

f (p) = f (0) cos p + cos(p 2) = cos 0 + cos 0 = 2 cos p = cos(p 2) = 1

Da devemos ent ao obrigatoriamente ter:

Por em, como somos estudiosos da trigonometria, sabemos que: cos p = 1 p = 2k1 cos(p 2) = 1 p 2 = 2k2 onde k1 e k2 s ao inteiros (nenhum dos quais e zero, pois p n ao e zero, j a que f e claramente n ao-constante) Dividindo ent ao estas duas u ltimas express oes, obtemos: p 2 2k2 k2 2= = = p 2k1 k1 e um n umero irracional Chegamos ent ao a um absurdo, pois sabemos que 2 Espero que voc es tenham gostado destes diversos problemas, e mais importante ainda, que tenham percebido as semelhan cas e diferen cas entre os problemas de olimp ada e os problemas de sala de aula. Penso tamb em que agora voc es estejam convencidos que uma pessoa n ao precisa ter estudado at e o terceiro ano da faculdade para fazer estes problemas. Eles s ao acess veis a todos. Est ao encerrados portanto os trabalhos deste cap tulo.

10

Cap tulo 2

Como montar um projeto Olimp ada de Matem atica na sua escola


Este cap tulo talvez seja o mais importante de todos. Desejo aqui passar a voc es algumas dicas de como implementar um projeto de Olimp ada em suas escolas, sejam elas p ublicas ou privadas. Para facilitar o entendimento das p aginas que se seguem, resolvi dividir o cap tulo em v arios t opicos, que listo a seguir: 1. O que e um projeto de Olimp ada? 2. Quais as vantagens de se ter um projeto como este na escola? 3. Que professores/alunos devem participar? 4. Como denir as turmas e os hor arios das aulas. 5. Material necess ario. 6. Sugest ao de programas did aticos. 7. Posso sofrer rejei ca o de alunos/professores da escola? 8. O que eu (professor) ganharei com isso? 9. O que meus alunos ganhar ao com isso? 10. Montando uma mini-biblioteca. 11. Contratempos que podem acontecer. 12. Como dar continuidade ao processo? 13. Est mulo constante aos seus alunos. 14. Estreitando os la cos com SBM e as universidades. Vamos ent ao discutir estes t opicos um a um. O que ser a dito aqui n ao pode ser tomado como certeza de sucesso futuro, s ao apenas algumas dicas que v ao servir como guia para que os interessados possam iniciar/continuar um trabalho semelhante. Vamos ao que interessa!

11

12

2.1

O que e um projeto de Olimp ada?

J a discutimos nesse livreto v arios t opicos relacinados com as competi co es matem aticas: seu hist orico, bases pedag ogicas, exemplos de problemas...mas o que vem a ser um projeto de Olimp ada de Matem atica na escola? Este projeto consiste na prepara ca o de alunos para competi co es de matem atica. Um grupo de professores de matem atica da escola vai se reunir e iniciar a prepara ca o de algumas turmas para as competi co es matem aticas. Ensinando o qu e? Obviamente, matem atica! Por em n ao uma aula comum, e nem um refor co escolar. Os t opicos a serem abordados s ao aqueles exemplicados no cap tulo anterior (vamos j a discutir mais sobre isso), de uma maneira mais informal, com o la co professor-aluno mais estreito, de modo que as pessoas que se re unam ali, possam desfrutar do simples prazer de discutir matem atica.

2.2

Quais as vantagens de se ter um projeto como este na escola?

O principal objetivo do projeto Olimp ada de Matem atica e desenvolver o ensino de matem atica na escola. Acreditamos que projetos como este s ao uma porta para o futuro, uma das alternativas de melhorar como um todo a educa ca o em nosso pa s. Como este processo funciona? O ensino de matem atica em sala de aula e nas aulas para olimp ada devem funcionar de maneira harmoniosa, um complementando o outro, de prefer encia at e ministrado pelos mesmos professores. O aluno que frequenta as aulas de olimp ada, vai ter oportunidade de estar em contato com novas id eias da matem atica, e isso certamante vai estimular seu racioc nio e sua criatividade. Rapidamente o seu rendimento escolar vai melhorar, n ao s o em matem atica, mas tamb em nas outras mat erias. Ele vai virar ent ao um referencial na sala, sendo respeitado pelos colegas, vai tirar d uvidas, etc... e isso vai fazer com que os pr oprios colegas se sintam estimulados a assistir as aulas de olimp ada tamb em. Com o passar do tempo o professor vai notar que o embasamento dos alunos melhorou, e ele mesmo vai se sentir mais estimulado a ensinar, a procurar coisas mais interessantes para passar aos alunos. Com isso ele vai tamb em estudar e aprender mais. E assim o ciclo recome ca com professores e alunos, ainda mais estimulados. Observe que o mesmo processo acontecer a se forem ofertadas aulas para olimp ada de qu mica, f sica, biologia, inform atica, astronomia... Por que voc e diz que a participa ca o do aluno na olimp ada de matem atica tende a melhorar seu rendimento em sala de aula, tanto em matem atica como nas outras disciplinas? um princ As competi co es de matem atica s ao em geral bem mais dif ceis que as provas do col egio. E pio simples: duas pessoas v ao participar de uma corrida de 3km. Um deles treina exatos 3km todo dia, enquanto o outro treina 5km. No dia da corrida quem vai sentir mais facilidade? Ao treinar para fazer uma prova mais dif cil, o seu rendimento nas provas do col egio vai certamente aumentar. E nas outras disciplinas? Bem, n ao e porque n os somos matem aticos, mas de certa forma todos devem concordar que matem atica e o portugu es desempenham um papel essencial no processo educacional. E se uma pessoa tem a oportunidade de desenvolver o racioc nio e o lado criativo, com certeza isso o ajudar a diretamente em outras ci encias exatas com f sica e qu mica, por exemplo. Os projetos de Olimp ada de Matem atica nas escolas s ao relativamente novos, t em no m aximo 10 anos, tanto em minha cidade (Fortaleza) como em S ao Paulo, que foram as pioneiras. O que houve na cidade de Fortaleza foi um grande salto da educa ca o local, que nos u ltimos anos vem conseguindo resultados expressivos a n vel nacional. Obviamente, n ao tenho a pretens ao de apontar a Olimp ada de Matem atica como u nico respons avel por isso, mas devo inclu -la, juntamente com os demais projetos de olimp ada cient cas, como fator importante para este salto de

13

qualidade. Seguem ent ao alguns exemplos concretos: Nos u ltimos anos, dentre os medalhistas em olimp adas cient cas (matem atica, f sica, qu mica,...) de n vel regional e nacional, h a sempre presen ca marcante de cearenses. Nos concursos e vestibulares mais concorridos do pa s, como por exemplo o ITA e o IME, a quantidade de estudantes de Fortaleza aprovados vem crescendo a cada ano. Para se ter uma id eia, no u ltimo vestibular do ITA (2003-2004), s o a cidade de Fortaleza aprovou uma quantidade de alunos compar avel (salvo engano maior) a todo o estado de S ao Paulo. Agora uma estat stica curiosa: nos u ltimos anos, em duas oportunidades (1994 e 2000) o primeiro lugar do vestibular para Medicina na UFC, foi um estudante que havia participado da Olimp ada Internacional de Matem atica. Em 1994, o estudante Marcondes Fran ca foi a IMO em Hong Kong e em 2000 o estudante Ulisses Medeiros foi a IMO na Cor eia. Ambos amigos de muito bom grado, hoje com bastante sucesso em suas carreiras m edicas. Resumindo, um diretor que implante essas id eias em sua escola pode esperar a m edio prazo um salto de qualidade na forma ca o de seus alunos, que ser a o reexo do desempenho deles nas competi co es culturais e vestibulares. Esses resultados se tornam com freq u encia, e merecidamente, motivo de marketing das escolas particulares de Fortaleza. E agora tamb em do Governo do Estado, que atrav es do Projeto Numeratizar iniciou no ano passado (2003) uma Olimp ada de Matem atica da Rede P ublica que reuniu 150.000 alunos, e tem tudo para come car a reerguer o ensino p ublico do nosso Estado.

2.3

Que professores/alunos devem participar?

A princ pio, todos os professores de matem atica interessados podem fazer parte do novo projeto de olimp ada da escola. Pode ser feita uma reformula ca o do hor ario de cada um, por exemplo um professor que tem 20 horas semanais em uma escola pode acertar com a dire ca o para ter 16 horas em sala de aula usual e 4 horas com turmas de olimp ada. Denidos aqueles professores que v ao dar aulas para a prepara ca o de alunos para a olimp ada, e necess ario que o material a ser abordado seja dividido entre eles (discutiremos sobre o material mais adiante) e que seja combinado quem vai ser respons avel por qual turma e em que hor ario (j a falaremos disso tamb em). E os alunos? Como devem ser escolhidos? Para uma escola que j a possui um projeto como este, ou escolas com grande quantidade de alunos, geralmente e feito um pequeno teste de sele ca o. Esta deve ser uma prova apenas para detectar os poss veis talentos que voc e professor vai ter em suas turmas. O ideal e que seja permitida a participa ca o de todos os alunos interessados, sem cortes iniciais. Pois a experi encia nos diz que uma turma de olimp ada que come ca por exemplo com 30 alunos, depois de um semestre tem apenas 20, depois de um ano, 15 ou menos. H a uma evas ao natural dos alunos. Por v arios motivos: Nem todo mundo vai querer car indo para a escola em hor arios extras. Muitos v ao olhar logo e dizer: -T o fora! J a sou muit ssimo ocupado (isso e o que ele pensa). Isso e coisa de nerd... Outros come cam e n ao aguentam o roj ao. Tenha em mente que nem todos que est ao l a ter ao voca ca o para matem atica (certamente ter ao para outras mat erias...). Alguns desses se esfor car ao e aprender ao muitas coisas, outros desistir ao. N ao que triste. Aviso aos professores: voc e ter a consigo v arios tipos de aluno. Sua turma pode ser bastante heterog enea inicialmente, mas n ao se preocupe, isso se resolver a com o tempo. Vale muito mais a pena trabalhar com alunos esfor cados e estudiosos do que com aqueles que se acham superiores e n ao s ao nada modestos (estes podem at e ser realmente inteligentes...), mas que s o cam atrapalhando a aula. Ou seja, ponha logo tudo nos eixos e dite as regras: que ali ningu em e melhor do que ningu em.

14

2.4

Como denir as turmas e os hor arios das aulas

A Olimp ada de Matem atica vem se tornando bastante competitiva ao longo dos anos. A cada ano mais pessoas v em participando, e como resultado disto a sele ca o e cada vez maior. N ao apenas por este motivo, o ideal e que um aluno comece a se preparar para fazer Olimp ada de Matem atica o mais cedo poss vel. Quando o aluno e mais novo, sua base ainda est a em forma ca o, e desta forma a prepara ca o ol mpica pode ajudar bastante no desenvolvimento de sua maturidade matem atica. Al em disso tem-se mais tempo para trabalhar com o(a) garoto(a). Quando pensamos em preparar um aluno j a mais velho (digamos 2o ou 3o ano do ensino m edio), devemos corrigir v arios defeitos de base que ele j a adquiriu ao longo da vida, para depois come car a ensinar alguns t opicos de matem atica ol mpica e a desenvolver as id eias. E ent ao, qunado voc e v e que o aluno tem potencial, ele termina o 3o ano, e n ao vai mais participar das competi co es sob sua tutela. portanto fortemente recomendado que voc E e comece o seu projeto de Olimp ada de Matem atica com turmas de 5a s erie at e o 1o ano. Se poss vel uma turma para cada s erie. Caso isso n ao seja poss vel, um outro modo seria dividir os alunos pelos n veis da OBM: n vel I (5a e 6a s eries), n vel II (7a e 8a s eries) e n vel III (ensino m edio). As aulas de olimp ada dever ao ser em hor arios extra-classe, ou seja em per odos diferentes das aulas usuais dos alunos. Por exemplo se os alunos da escola assitem aula pela manh a, voc e pode marcar as aulas de olimp ada duas vezes por semana ` a tarde de 2:30 as 4:10. Ou se voc e tem alunos em dois turnos, manh a e tarde, e quer reuni-los numa s o turma de olimp ada, isso pode ser feito duas vezes por semana ` a noite (6:30 - 8:00), ou aos s abados. De fato, os hor arios v ao se acomodar de acordo com a necessidade e disponibilidade dos alunos e professores envolvidos. Agora quantas aulas deve ter cada turma de olimp ada? Bem essa e uma pergunta ex vel. Acho que em 4 horas semanais (2 aulas de 2 horas) para cada turma j a se pode desenvolver um bom trabalho. Se o seu col egio j a vem com uma turma que passou pela 5a, 6a, 7a e agora est a na 8a s erie, voc e pode pensar em aumentar para 6 horas semanais (isso vale para turmas experientes). Caso a situa ca o esteja economicamente dif cil, tamb em podemos pensar em 4 horas semanais por n vel. O que daria um custo inicial para a escola de 8 aulas se pensarmos em come car s o com n veis I e II. Agora, o importante bater o centro. Depois as coisas v e come car!! E ao se acertando, voc e p oe mais umas aulas ali, d a um intensiv ao perto das provas, faz um cursinho nas f erias... Outra dica importante. Se voc e for optar por dar 4 ou 6 aulas em uma turma, de prefer encia fa ca isso em dias intercalados (seg-qua-sex ou ter-qui ou qua-s ab). Intercalando os dias eles v ao ter mais contato com voc e, al em do que, voc e pode explorar mais o enorme potencial que t em os exerc cios pra casa, e pass a-los duas ou tr es vezes por claro, como j semana. E a disse antes, isso vai depender da disponibilidade de tempo/dinheiro de todos.

2.5

Material necess ario

Agora chegamos a um ponto crucial. Voc e, professor deve tomar alguns livros e materiais como base para treinar seus alunos. Surpreenda-se com as coisas novas que voc e mesmo vai aprender! A maior fonte de material e sabedoria dos tempos modernos chama-se Internet. H a diversos sites que disponibilizam muito material para o treinamento ol mpico. E o mais importante: de gra ca. Basta entrar num site de busca e se aventurar. Abaixo falaremos um pouco dos t opicos a serem estudados para a Olimp ada de Matem atica, os livros recomendados para iniciantes e alguns sites interessantes. Tradicionalmente, os assuntos abordados s ao divididos em 4 temas: Teoria dos N umeros, Algebra, Geometria e Combinat oria. Falaremos um pouco de cada um deles, indicando alguns livros e ainda apresentando uma sugest ao de conte udo a ser trabalhado.

15

2.5.1

Teoria dos N umeros

Os t opicos em Teoria dos N umeros s ao aqueles relacionados com as propriedades elementares dos n umeros inteiros: divisibilidade, n umeros primos, fatora ca o, MDC, MMC,... Este e um t opico cl assico na olimp ada, e fortemente recomendado para iniciar o treinamento de jovens, pois v arios destes assuntos j a s ao abordados no col egio. Livros recomendados para iniciantes: Teoria Elementar dos N umeros, Edgar de Alencar Filho (Brasil) Aritmetica, Maria Elena Becker, Ed. Red Olimpica (Argentina) Aritmetica, Saulo Aranda (Venezuela) Livro recomendado para n vel Intermedi ario: Introdu ca o ` a Teoria dos N umeros, Jos e Pl nio Santos, Cole ca o Matem atica Universit aria (Brasil) N ao e obrigado o estudo por um destes livros acima, qualquer livro que cubra os mesmos assuntos pode ser utilizado como base.

2.5.2

Algebra

Nesta area, os alunos devem aprender bem as opera co es com n umeros e suas propriedades. A algebra estuda principalmente as equa co es, de primeiro e segundo graus, os sistemas de equa co es, os produtos not aveis. Para um aluno do ensino m edio, quando falamos em algebra, englobamos diversos assuntos: polin omios, n umeros complexos, seq u encias... A algebra e bastante ensinada no col egio. Da , o primeiro passo do aluno ser a compreender bem as li co es da escola, para depois vir a se aprofundar nas aulas de olimp ada. Livro recomendado para iniciantes: Livros do col egio em geral. Livro recomendado para n vel intermedi ario: Challenging Problems in Algebra, I.M. Yaglom (EUA) Veja mais sobre algebra na se ca o: T ecnicas de Resolu ca o de Problemas.

2.5.3

Geometria

a cl Aqui todos n os sabemos do que se trata. E assica geometria plana. Sempre presente nas provas de olimp ada. Tamb em e bastante ensinada em sala de aula, de modo que o aluno deve saber bem a mat eria do col egio para depois se aprofundar. Livro recomendado para iniciantes: Matem atica Elementar, vol. 9, Gelson Iezzi et al. (Brasil) Livros recomendados para n vel intermedi ario: Geometria, vols 1 e 2 , A.C. Morgado (Brasil) Challengig Problems in Geometry , I.M. Yaglom (EUA) Geometry Revisited, H.S. Coxeter (EUA)

16

2.5.4

Combinat oria

D a-se o nome de combinat oria a tudo aquilo que n ao e teoria dos n umeros, algebra ou geometria. Os problemas de combinat oria s ao aqueles leg timos de olimp ada, que usam bastante a criatividade do aluno. Na lista de t opicos a serem estudados merecem destaque: paridade, contagem, princ pio das gavetas, jogos, invariantes... Este assunto e a grande novidade. Geralmente os estudantes n ao t em contato com este tipo de matem atica na escola, por isso os professores devem ir bem devagar e sempre estimulando o racioc nio dos seus alunos. Livros recomendados para iniciantes: Mathematical Circles, the russian experience, D. Fomin (Russia - traduzido para o ingl es) Contos com contas, Miguel de Guzman (Portugal) Aventuras Matem aticas, M. Guzman (Portugal) Livro recomendado para n vel intermedi ario Combinat oria e Probabilidade, A.C. Morgado et al (Brasil)

2.5.5

T ecnicas de Resolu c ao de Problemas

Os alunos que se preparam para as olimp adas de matem atica devem ter seu treinamento voltado para a resolu ca o de problemas. Um grande amigo, Rui Lopes Viana Filho, medalha de ouro na Olimp ada Internacional de Matem atica em 1998, costumava dizer que seu treinamento matem atico era baseado na resolu ca o de exerc cios. Havia o que ele chamava de sua caixa de ferramentas que eram aqueles problemas-chave que ele j a havia resolvido. Ent ao quando era chegada a hora das provas, de resolver novos problemas, ele sempre recorria ` a sua caixa de ferramentas para ver mais ou menos assim que as coisas funcionam. se encontrava alguma id eia parecida. E Alguns livros recomendados para o Olimp ada de Matem atica s ao colet anias de problemas e solu co es. Estes abordam todos os assuntos supracitados e s ao de fundamental import ancia para os professores (que podem fazer listas, simulados, aulas de problemas...) e para os alunos (que podem tentar resolv e-los e caso n ao consigam, podem aprender novas t ecnicas entendendo as solu co es). Abaixo listamos v arios destes livros. Livros recomendados para iniciantes: Olimp adas de Matem atica do Estado do Rio de Janeiro, Eduardo Wagner et al (Brasil) Olimp adas Paulistas de Matem atica, 1o grau (Brasil) Olimp adas Paulistas de Matem atica , 2o grau (Brasil) Contest Problem Book (AHSME e AIME), volumes 1 a 6 (EUA) Livros recomendados para n vel intermedi ario: The Hungarian Problem Book I, II, III (Hungria) Olimp adas Brasileiras de Matem atica 1a a 8a (Brasil) Olimp adas Brasileiras de Matem atica 9a a 16a (Brasil) Olimp adas de Matem atica do Cone Sul, 1a a 4a (Argentina) Olimp adas de Matem atica do Cone Sul , 5a a 12a (Brasil) Olimp adas Iberoamericanas de Matem atica 1a a 10a (Brasil)

17

2.5.6

Revista Eureka!

Em 1998, com a reestrutura ca o da Olimp ada Brasileira de Matem atica, foi criada a revista Eureka!. Esta e uma publica ca o trimestral, que atualmente est a em seu volume 19, cujo principal objetivo e o treinamento brasileiro para as olimp adas de matem atica. Esta revista e extremamente recomendada para um projeto de olimp ada na escola. Para saber mais informa co es de como assinar e como pedir os n umeros anteriores consulte o site ocial da OBM: www.obm.org.br

2.5.7

Livros para estudos avan cados

Abaixo listamos mais alguns livros, agora mais avan cados, voltados para o treinamento ol mpico. Estes s ao recomendados apenas para alunos do ensino m edio que j a v em em prepara ca o constante e para professores. Todos s ao escritos em ingl es. 1. Olimp adas Internacionais de Matem atica 1959-1974 (EUA) 2. Olimp adas Internacionais de Matem atica, 1975-1980 (EUA) 3. Olimp adas Internacionais de Matem atica, 1981 - 1994 ( India) 4. Principles and Tecniches in Combinatorics, Cheng C. Chong (Taiwan) 5. Winning Solutions , Cecil Russeau and E. Lozanski (EUA) 6. 102 Problems in Combinatorics , Titu Andreescu (EUA) 7. Mathematical Miniatures , T. Andreescu e S. Savchev (EUA) 8. Mathematical Contests, todos os 7 volumes (95, 95-96, 96-97, 97-98, 98-99, 99-00, 00-01) 9. 101 Problems in Algebra, T. Andreescu (Australia) 10. Advanced Euclidean Geometry, M. Johnson (EUA) 11. Geometry Revisited, H.S. Coxeter et al (EUA) 12. USA Mathematical Olympiads, M. Klamkim (EUA) 13. The Tournament of the Towns, todos os volumes (Australia) 14. Combinatorics, Ioan Tomescu (Romenia) 15. Problems in Combinatorics and Graph Theory, I. Tomescu (Romenia) 16. Polynomials, E. Barbeau (EUA) 17. Complex Numbers in Geometry , S. Zahn (Cor eia) 18. Mathematical Gems, volumes 1,2,3. H. Honsberger (EUA) 19. Mathematical Morsels, H. Honsberger (EUA) 20. Mathematical Mosaics , R. Vakil (Canad a) 21. In P olyas Footsteps, (EUA) 22. Problem solving through problems, I. Larson (EUA) 23. Problem Solving Strategies, A. Engel (Alemanha) 24. Geometric Inequalities , O. Botemma (EUA) 25. The Putnam Competition, volumes 1,2,3 (EUA) 26. Introduction to the Theory of Numbers, Niven and Zuckerman (EUA) 27. Elementary Number Theory, E. Burton (EUA)

18

2.5.8

Buscando na Internet

Abaixo est ao listados alguns sites bastante interessantes. Nestes sites voc e pode encontrar informa co es sobre a olimp ada de matem atica, provas de anos anteriores, material de treinamento, revistas, fotos e v arias outras coisas. A Internet e uma fonte inesgot avel de conhecimento e informa ca o. Certamente os alunos v ao se empolgar muito mais se puderem ver fotos, listas de medalhados, curiosidades... E se eles pr oprios puderem fazer download de provas e material de treinamento, melhor ainda! www.obm.org.br - e a p agina ocial da Olimp ada Brasileira de Matem atica. Cont em tudo que voc e precisa saber e ainda com in umeros links para os sites ociais das Olimp adas Estaduais. www.sbm.org.br - e a p agina ocial da Sociedade Brasileira de Matem atica, onde voc e pode se cadastrar e ganhar in umeras vantagens como por exemplo descontos na compra de livros. www.oma.org.ar - p agina da Olimp ada de Matem atica Argentina. www.numeratizar.mat.br - p agina do projeto Numeratizar, que e a Olimp ada da Escola P ublica do Cear a, com material de treinamento elementar. www.opm.mat.br www.impa.br - Olimp ada Paulista com v arias coisas legais - Grupo Teorema de Matem atica, tamb em com in umeras coisas legais.

www.teorema.mat.br

- Site do IMPA - Instituto de Matem atica Pura e Aplicada. - Olimp ada Peruana de Matem atica (esse link est a na OBM)

http://www.geocities.com/CollegePark/7174 www.google.com.br desejem.

- site de busca: deve ser o mais utilizado por voc es para procurar qualquer coisa que

2.6

Sugest ao de programas

No Ap endice A do livro, damos sugest oes de programas para o treinamento ol mpico. Os professores devem adequar estas sugest oes ` a sua disponibilidade de tempo, de livros e ao rendimento dos seus alunos. Os conte udos v ao desde a 4a s erie at e o 2o ano do ensino m edio. Era um pouco grande para ser inclu do aqui, no meio do texto, por isso, resolvi deixar como anexo.

2.7

Posso sofrer rejei c ao de alunos/professores da escola?

Toda novidade traz consigo admira ca o de uns, rejei ca o de outros. S ao os dois lados da moeda. Experi encias comprovam que pode haver certa rejei ca o ao programa de treinamento ol mpico na sua escola. Tamb em n ao h a motivo para desespero, pois s ao situa co es perfeitamente contorn aveis. Certas fac co es de alunos podem n ao querer participar do seu projeto. Natural. Op ca o deles. Se eles insistirem em tratar seus alunos como CDF, bitolados, nerds... n ao se preocupe, isso e normal e vai passar com o tempo. Em certos estados, isso reete o pensamento da juventude em geral, de que ser estudioso e esfor cado e errado, e coisa de extraterrestre, enquanto o legal e ser relaxado e passar se arrastando, mas ser um cara da galera. Fico triste com tais atitudes, mas ainda assim acredito que esta mentalidade v a mudar ao longo do tempo. Em meu estado, o Cear a, os alunos esfor cados e inteligentes, s ao muito estimados por seus colegas e por suas escolas, alguns at e com certas regalias. O que mais me preocupa e a rejei ca o que pode haver por parte da dire ca o e de outros professores da escola. Voc e pode se perguntar: Ora por que essas pessoas v ao querer atrapalhar? Eu diria que e comodismo ou inveja da parte delas. S ao aquelas pessoas que n ao querem mudar, que acham que

19

tudo est a bom como est a, e quando aparece uma pessoa propondo uma novidade para desenvolver as coisas, elas cam desencorajando: isso n ao vai dar certo, melhor nem come car,... O que fazer com essas pessoas? Bem, o melhor e dizer: se voc e n ao quer ajudar, tudo bem, mas por favor tamb em n ao atrapalhe! Desista de convencer todo mundo que a sua id eia e boa. Voc e n ao vai conseguir. S o come cando e passando algum tempo e que voc e poder a mostrar, atrav es do seu trabalho, como aquilo valeu a pena. Acima de tudo: n ao se preocupe, essas vozes opositoras v ao caindo ao longo do tempo. Ap oie e d e cr edito tamb em aos outros projetos da escola: caminhada ecol ogica, grupo de dan ca, grupo de teatro, ONGs,... pois assim como o seu, todos eles s ao importantes! Fa ca como acima, se n ao quiser ajudar, tamb em n ao atrapalhe. Fique de olho tamb em na parte dos educadores que dizem que a Olimp ada de Matem atica e um processo exclu claro que alguns alunos v E! Onde j dente. NAO a se viu tamanha tolice, me perdoem as palavras! E ao se dar melhor que outros, mas isso e muito natural. Obviamente estes outros ser ao muito bons em outras areas, talvez artes, m usica, isso que a Olimp esportes... basta que lhes sejam dadas as oportunidades. E ada de Matem atica, entre outras coisas, faz: d a oportunidade aos alunos para desenvolverem seu potencial. Os educadores que pensam na Olimp ada de Matem atica como fator de diferencia ca o social e exclus ao s ao os mesmos que desejam que os alunos sejam tratados como iguais: sem oportunidade e nem perspecticas para nenhum. Vamos mudar esse quadro. Vamos dar chances, sonhos e rumos a quem quiser segui-los. Que bom se isso pudesse ser feito em todos os ramos, n ao s o em matem atica!

2.8

O que eu (professor) ganharei com isso?

Os professores que participarem do projeto de Olimp ada de Matem atica da escola s o t em a ganhar. Um grande presente(!?) que eu considero e a mudan ca da sua rotina, pois agora voc e vai ter que preparar novas aulas, com n vel de diculdade maior do que o da sala de aula comum. Certamente ser a mais dif cil no come co, mas podem ter certeza que e muito recompensador. As aulas de olimp ada n ao precisam ser t ao formais e r gidas como as aulas do hor ario de classe. Com isso, o professor se aproxima mais dos alunos e se torna realmente amigo deles. Ao longo do tempo a aula vai se tornando cada vez mais um bate-papo divertido entre amantes de matem atica. Outro b onus que eu aponto e que voc e, professor, vai enriquecer ainda mais seu conhecimento matem atico. Ao preparar e ministrar sua aulas de olimp ada, certamente voc e vai se deparar com coisas novas, e vai tamb em aprender bastante. Todos n os temos algo a aprender: os alunos conosco e n os com eles.

2.9

O que meus alunos ganhar ao com isso?

Os alunos que participarem ter ao ainda mais recompensas. J a disse anteriormente que a olimp ada far a com que o aluno estimule seu racioc nio e isso vai possibilitar um desenvolvimento educacional melhor, n ao s o em matem atica, como tamb em nas outras areas. Al em disso, na olimp ada se faz muitos amigos. Pessoas de outras escolas, at e de outros estados, com interesses parecidos. Se algum deles tiver o prazer de se classicar na OBM, por exemplo, e vir a participar de um encontro como a Semana Ol mpica, ser a uma experi encia inesquec vel para o garoto. Garanto que ele ser a estimulado em 100% para continuar estudando. A Olimp ada de Matem atica tende a crescer no pa s, e isto passa a ser tamb em uma carta de recomenda ca o para o estudante adiante. Premia co es na Olimp ada ser ao importantes diferenciais na vida acad emica, quando o aluno for concorrer a bolsas na universidade. N ao s o a bolsas, mas tamb em a qualquer tipo de emprego. Um diploma de olimp ada, assim como um diploma de curso de ingl es ou inform atica, passa a fazer parte do curr culo do aluno.

20

Aqui no Cear a, este ano, os 363 alunos classicados na Olimp ada de Matem atica da Escola P ublica est ao recebendo o governo tomando uma bolsas mensais no valor de R$72, 00. J a pensou o quanto isso signica para fam lias carentes? E iniciativa pioneira, louv avel, de premiar quem estuda. Esses alunos est ao ainda recebendo treinamento em matem atica e certamente no futuro difundir ao seus conhecimentos e estar ao aqui para falar dos benef cios que a matem atica lhes trouxe.

2.10

Montando uma mini-biblioteca

Com o passar do tempo e arma ca o do projeto de olimp ada na escola, e importante a implanta ca o de uma minibiblioteca de matem atica. Esta pode ser apenas uma estante na biblioteca da escola, ou um arm ario na sala do professor, n ao importa. O que importa e que livros como os que foram citados acima (e por que n ao tamb em o material did atico usado nas aulas?) sejam disponibilizados para os alunos. A escola n ao precisa adquirir todos os livros de uma vez, basta comprar 1 ou 2 por m es, que em 3 anos sua minibiblioteca vai estar excelente! O acesso a ela pode ser apenas ` as pessoas interessadas na olimp ada, atrav es de carteirinha, chave do arm ario, etc, ou se preferirem a qualquer pessoa do col egio. Tamb em n ao precisa dizer que um cuidado com os livros e essencial. O rendimento dos alunos vai melhorar consideravelmente se eles puderem, de vez em quando levar um livrinho para casa...

2.11

Contratempos que podem acontecer

Tudo bem. Voc e j a iniciou seu projeto, t em alunos ass duos em v arias turmas de olimp ada, competindo a n vel municipal, estadual e nacional. J a venceu a rejei ca o inicial de alguns alunos e cabe cas da dire ca o. O que pode dar de errado agora? Eu diria que sua base j a est a s olida. Agora e s o tocar pra frente, tomando cuidado com poss veis contratempos. Vou chamar a aten ca o para dois poss veis acontecimentos. natural que voc E e tenha turmas melhores que outras, mas nem por isso concentre toda a sua aten ca o em uma importante o uxo cont turma s o. Esteja sempre olhando a forma ca o dos alunos que v em a seguir. E nuo, ou seja, formar bem os alunos da 8a s erie, os da 7a, os da 6a, os da 5a. Pois no outro ano tudo recome ca. Ocorreu na escola onde trabalhei aqui em Fortaleza. T nhamos uma turma muito boa, com uns 10 alunos excelentes, diversas vezes premiados. Todos os professores queriam dar aula para esses caras e meio que esqueciam os demais. Resultado: quando essas turma concluiu o col egio, havia s o 1 estudante de olimp ada na turma da s erie anterior. Fez-se um hiato, que s o conseguimos reparar 1 ou dois anos depois. Desde ent ao trabalhamos com aten ca o para com todas as s eries. Outro aspecto relevante e sempre conscientizar seus alunos que a matem atica que eles aprendem na olimp ada e diferente da matem atica da sala de aula. Dizer aos alunos que eles n ao devem esnobar colegas ou professores. Humildade sempre. Na maioria dos casos, os alunos nem pensam em esnobar algu em (a n ao ser seu irm ao mais novo...), mas de vez em quando aparecem uns espertinhos, a voc e deve baixar a bola deles.

2.12

Como dar continuidade ao processo?

J a foi dito algo sobre isso anteriormente. Sobre a aten ca o para com todas as turmas, para que n ao haja buracos. Sobre a minibiblioteca. Esses s ao alguns fatores importantes na continuidade do processo. aconselh E avel que os professores se re unam de vez em quando, para avaliar o progresso dos seus estudantes e os rumos a serem tomados. Divis ao dos conte udos entre os professores tamb em e muito importante. Para que cada um

21

saiba suas obriga co es e seus objetivos. Outros t opicos relevantes para dar prosseguimento ao seu projeto ol mpico s ao: o est mulo constante aos seus alunos e professores, e tamb em o contato com a SBM, OBM e universidades. Estes ser ao discutidos em separado nas se co es seguintes.

2.13

Est mulo constante aos seus alunos

Chegamos a um t opico crucial. Como manter seus alunos estimulados? O pr oprio clima de amizade presente na aula de olimp ada j a e um bom est mulo para os garotos. Vale tamb em de vez em quando distribuir uns pr emios, fazer umas gincanas na sala de aula, sempre promovendo a competi ca o matem atica saud avel entre eles. A minibiblioteca e o acesso ` a Internet s ao sem d uvida armas poderosas no est mulo a seus alunos. Atrav es destas o estudante se sentir a cada vez mais seguro para pesquisar e com isso passar a rapidamente a desenvolver seu conhecimento matem atico por conta pr opria! Com o passar do tempo, eles (os alunos) v ao come car a participar das competi co es de matem atica. A OBM e a Olimp ada Estadual talvez sejam dif ceis no come co e e prov avel que voce n ao veja seus alunos se classicarem na primeira vez. Nada de des animo! Uma boa conversa com eles, para que eles possam assimilar o rev es, vai cair bem. Seria interessante que a escola organizasse uma olimp ada interna, fazendo uma festinha e congregando pais, alunos e professores. Distribua v arias medalhas e pr emios, assim os garotos v ao car felizes...(seus pais tamb em!!!). Voc e pode tamb em conversar com outros professores da sua cidade, e montar uma olimp ada do seu munic pio. Competindo na esfera municipal, todos os alunos ter ao mais chances de premia ca o. Trabalhando nessas competi co es menores, eles v ao se sentir cada vez mais seguros e capazes para concorrer a n vel estadual e nacional. E o mais importante, estudar ao sempre com mais dedica ca o. Assim voc e estar a no caminho certo.

2.14

Estreitando os la cos com SBM e as universidades

bastante recomendado que o professor se associe ` E a SBM, pois isso garante uma s erie de vantagens: descontos para comprar livros, para receber a Revista do Professor de Matem atica, para participar de eventos. A SBM, atrav es da OBM e a principal apoiadora da Olimp ada de Matem atica no nosso pa s, estimulando os alunos atrav es de listas de tamb discuss ao via internet e da revista Eureka. E em a SBM/OBM que contribui com parte da verba destinada para a realiza ca o das competi co es regionais, que v em crescendo ao longo do tempo. Hoje em dia, a maioria dos estados brasileiros t em sua pr opria Olimp ada de Matem atica. Al em disso, o contato com a Universidade tamb em deve ser estreito, pois n ao esque camos que esta deve ser o objetivo importante que os professores do ensino m para os nossos alunos. E edio e fundamental possam trocar id e as com os mestres e doutores do ensino superior e ter acesso ` as bibliotecas (isso e legal!!). Com o passar do tempo, a universidade pode criar programas de ver ao para jovens interessados na olimp ada, como j a e feito no Rio e no Cear a, por exemplo. H a jovens talentos que, no IMPA(RJ), iniciam seus estudos de mestrado, e em seguida doutorado, ainda adolescentes e se tornam excelentes matem aticos. N ao tenha d uvida que a universidade vai dar muito apoio a esses jovens talentos, e eles s o ter ao a agradecer.

22

Cap tulo 3

Listas de Exerc cios


Este cap tulo foi idealizado com o objetivo de mostrar mais alguns problemas do interessante mundo novo da Matem atica. S ao 10 listas, cada uma com 5 problemas, recomendadas para alunos e professores iniciantes. Voc e pode us a-las como bem entender com seus pupilos. Uma dica: passe uma lista como essas (4 ou 5 problemas) uma vez por semana para eles. Estes s ao problemas que cobrem os mais variados t opicos de matem atica para um aluno iniciante. Existe uma certa ordem de diculdade nas listas. Voc e ver a que os problemas se tornam cada vez mais dif ceis (mas s o um pouquinho...). Boa sorte!

Lista 1
Problema 1 Colocar em cada um dos pontos marcados sobre os lados do tri angulo os n umeros de 1 a 9, de modo que a soma dos quatro n umeros colocados sobre cada lado seja a mesma.

Problema 2 Um pai tem 40 anos e a soma das idades de seus tr es lhos e 22 anos. Dentro de quantos anos a idade do pai ser aa soma das idades dos lhos? Problema 3 Em um ano n ao bissexto, qual e o dia do meio? (Isto e, aquele cujo n umero de dias anteriores a ele e igual ao n umero de dias posteriores a ele) Problema 4 Temos tr es caixas (em la) e tr es frutas: um abacaxi, uma banana e um caj a. Cada caixa cont em uma fruta e sabe-se que: A caixa verde est a` a esquerda da caixa azul. A caixa vermelha est a` a direita da banana. 23

24

O caj a est a` a direita da caixa vermelha. Qual fruta est a em qual caixa? Problema 5 Raul e Cida formam um estranho casal. Raul mente ` as 4as, 5as e 6as feiras, dizendo a verdade no resto da semana. Cida mente aos domingos, 2as e 3as feiras, dizendo a verdade nos outro dias. Certo dia ambos declaram: amanh a e dia de mentir. Em que dia foi feita esta declara ca o? (Olimp ada Carioca/93)

Lista 2
Problema 6 Bruce Willis se meteu numa tremenda enrascada: est a em uma fonte, na qual est a armada uma bomba que vai explodir em alguns minutos. Se ele tentar sair da fonte, tudo vai pelos ares... mas ele tem uma chance: conectada ` a bomba existe uma balan ca, e esta balan ca desarma a bomba quando se p oem exatamente 4 litros de agua sobre ela. Bruce Willis tem 2 recipientes: um com capacidade para 3 litros e outro com capacidade para 5 litros, como ele deve fazer para sair vivo dessa fria? (baseado numa cena do lme Duro de matar III). Problema 7 poss Bruno falou: Antes de ontem eu tinha 10 anos, mas completarei 13 ano que vem. E vel que Bruno esteja falando a verdade? Problema 8 Augusto possui uma grande quantidade de 0 s, 1 s, 3 s, 4 s, 5 s, 6 s, 7 s, 8 s e 9 s, mas disp oe somente de vinte e dois 2 s. At e que p agina ele poder a numerar as p aginas de seu novo livro? (Olimp ada Carioca - 94) Problema 9 Um crime e cometido por uma pessoa e h a quatro suspeitos: Andr e, Eduardo, Rafael e Jo ao. Interrogados, eles fazem as seguintes declara co es: 1. Andr e: Eduardo e o culpado. 2. Eduardo: Jo ao e o culpado. 3. Rafael: Eu n ao sou culpado. 4. Jo ao: Eduardo mente quando diz que eu sou culpado. Sabendo que apenas um dos quatro disse a verdade, quem e o culpado? Problema 10 (i) Mostre como dispor os n umeros 1, 2, 3, 4, 5, 6, 7, 8, 9 nas casas de um quadrado 3 3 para formar um quadrado m agico, ou seja, para que a soma dos n umeros nas linhas, colunas e diagonais seja a mesma. (ii) Prove que em todo quadrado m agico o n umero da casa central deve ser o 5. (iii) Voc e seria capaz de p or os n umeros de 1 a 16 (sem repeti ca o) em um quadrado 4 4 de modo a formar um quadrado m agico? Lembre-se que a soma dos n umeros em cada linha, coluna e nas duas diagonais deve ser igual.

25

Dica absurdamente boa: Tente primeiro mostrar que a soma comum (de cada linha, coluna ou diagonal) deve ser 34.

Lista 3
Problema 11 Sobre uma mesa h a 21 garrafas de guaran a, das quais 7 est ao cheias, 7 est ao vazias e 7 est ao cheias at e a metade. Reparta-as entre tr es garotos de modo que os tr es recebam a mesma quantidade de garrafas e de guaran a. Obs: N ao vale derramar o guaran a de uma garrafa em outra. Problema 12 No barril A h a 100 litros de agua e no barril B h a 100 litros de alcool. Retira-se 1 litro de agua de A e coloca-se em B . Em seguida retira-se 1 litro da mistura de B e coloca-se em A. No nal das opera co es, h a mais agua no alcool ou mais alcool na agua? Problema 13 Chamam-se pal ndromos os n umeros inteiros positivos que n ao se alteram quando e invertida a ordem de seus algarismos (por exemplo: 383, 4224, 74847...). Qual e o n umero total de pal ndromos de cinco algarismos? Problema 14 (OBM/98) Pintam-se de preto todas as faces de um cubo de madeira cujas arestas medem 10 cent metros. Por cortes paralelos ` as faces, o cubo e dividido em 1000 cubos pequenos, cada um com arestas medindo 1 cent metro. Determine: (a) o n umero de cubinhos que n ao possuem nenhuma face pintada de preto. (b) o n umero de cubinhos que possuem uma u nica face pintada de preto. (c) o n umero de cubinhos que possuem exatamente duas faces pintadas de preto. (d) o n umero de cubinhos que possuem tr es faces pintadas de preto. Problema 15 Considere a sequ encia (1, 2, 2, 3, 3, 3, 4, 4, 4, 4, 5, 5, 5, 5, 5...), cujos termos s ao os inteiros consecutivos em ordem crescente, e na qual o inteiro n ocorre n vezes. Ache o resto da divis ao por 5 do 1999 termo. (Olimp ada Carioca 93)

Lista 4
Problema 16 Escrevemos a lista dos n umeros naturais 1, 2, 3, 4, 5, 6, 7, 8, 9, 10, 11, 12, 13, .... Qual e o d gito que ocupa o 2004 lugar? Problema 17 Na soma IRA + ARO + ORA, as letras I, R, O, A representam os d gitos 1, 3, 8 e 9 (d gitos distintos para letras distintas, n ao necessariamente nessa ordem). (a) Qual e a maior soma poss vel? (b) E a menor? Problema 18

26

(a) Uma pe ca e colocada em uma extremidade (do lado de fora) de uma la 1 21 de casas. Cada jogador na sua vez pode mover a pe ca uma ou duas casas para a frente (s ao dois jogadores). Ganha o jogador que atingir a u ltima casa do tabuleiro. Quem possui a estrat egia vencedora? (b) Generalize para o caso de o tabuleiro ser 1 N . Problema 19 (Rioplatense/2003) Algumas pessoas v ao a uma pizzaria. Cada pessoa que est a faminta quer comer 6 ou 7 peda cos de pizza, e as outras querem comer 2 ou 3 peda cos. Cada pizza possui 12 peda cos. Sabe-se que 4 pizzas n ao s ao sucientes para satisfazer a todos, mas com 5 pizzas sobrariam alguns peda cos. Quantas pessoas foram ` a pizzaria? Quantas delas estavam famintas? Problema 20 Temos uma cartolina em forma de tri angulo equil atero. (a) Mostre como dividir a cartolina em 22 tri angulos equil ateros menores (n ao necessariamente do mesmo tamanho). (b) Mostre como dividir a cartolina em 17 tri angulos equil ateros menores (n ao necessariamente todos do mesmo tamanho).

Lista 5
Problema 21 Quando 1094 94 e desenvolvido, qual e a soma de seus algarismos? Problema 22 Daniel trocou de lugar as teclas da calculadora de Onofre da seguinte maneira: Calculadora de Onofre 7 4 1 8 5 2 0 9 6 3 Calculadora com as teclas trocadas 3 2 1 6 9 5 8 4 7 0

Sem notar que as teclas foram trocadas, Onofre multiplica dois n umeros de dois d gitos cada um e aparece como resultado 1722. Sabe-se que o verdadeiro resultado seria um n umero de tr es algarismos. Que n umeros Onofre queria multiplicar ? Problema 23 Em um tri angulo is osceles ABC (AB = AC , BAC = 30 ) marcamos um ponto Q no lado AB e um ponto P na mediana AD, de modo que P C = P Q (Q = B ). Ache P QC . Problema 24 (Bielor ussia/01) Encontre o menor n umero poss vel de elementos do conjunto: {1, 2, 3, ..., 28} que devem ser deletados para que o produto dos restantes seja um quadrado perfeito.

27

Problema 25 Vanessa escreve oito n umeros em cada um dos v ertices de um cubo. Em cada aresta escreve a soma dos dois n umeros poss dos v ertices adjacentes, e em cada face escreve a soma dos quatro n umeros das arestas adjacentes. E vel que a soma de todos os n umeros escritos no cubo seja 2004?

Lista 6
Problema 26 Tr es amigos planejam uma viagem ao campo de Rodrigo. Evaldo diz : Para chegarmos demoraremos no m aximo 5 horas. Andr ea ent ao fala: Garanto que chegamos em mais de 5 horas. Raul diz: A viagem dura mais de 2 horas. Se apenas um deles tem raz ao, qual deles e? O que se pode dizer sobre a dura ca o da viagem? Problema 27 Usando os algarismos 1, 2, 3, 4, 5, 6, 7, 8 e 9, cada um uma u nica vez, forme 3 n umeros de tr es algarismos de modo que o segundo n umero seja o dobro do primeiro e que o terceiro seja o triplo do primeiro. Problema 28 (OBM/99) Na gura, os tri angulos ABC e EGF s ao equil ateros. O per metro do tri angulo ABC e 132cm e, al em disso, AE = EC , BD = DC , EF = F C e DG = GE . B

A (a) Qual e o per metro da area sombreada?

(b) Que fra ca o da area do tri angulo ABC representa a area sombreada?

Problema 29 O ret angulo da gura foi dividido em 9 ret angulos menores, cujas areas est ao indicadas: 4 5 Determine o valor da area X . Problema 30 Z e guarda suas gurinhas em envelopes. Algumas gurinhas s ao quadradas e outras redondas. Em cada envelope ele p oe apenas gurinhas do mesmo tipo. Os envelopes continham 9, 15, 16, 17, 26 e 34 gurinhas. Ontem Z e deu um envelope de presente para seu irm ao e agora o n umero de gurinhas redondas que tem e o dobro do n umero de 2 3 1 X

28

gurinhas quadradas. Quantas gurinhas havia no envelope que ele deu? Qual o tipo de gurinha que h a em cada envelope restante?

Lista 7
Problema 31 Uma caixa cont em 100 bolas, das quais 30 s ao vermelhas, 30 s ao azuis, 30 s ao verdes e das 10 restantes, algumas s ao pretas e outras s ao brancas. Qual o n umero m nimo de bolas que devem ser retiradas da caixa, sem lhes ver a cor, para termos a certeza de que entre elas existem pelo menos 10 bolas da mesma cor? E para garantir que existem pelo menos 15 bolas da mesma cor? Problema 32 Quantos n umeros de 100 at e 999 possuem todos os seus algarismos distintos e ordenados de maneira crescente? Problema 33 As diagonais AC e BD do quadril atero ABCD se encontram no ponto O. Os per metros dos tri angulos ABC e ABD s ao iguais, assim como os per metros dos tri angulos ACD e BCD. Prove que AO = BO. Problema 34 S ao constru dos exteriormente ao Problema 35 Uma escola tem exatamente 100 arm arios e 100 estudantes. No primeiro dia de aula os estudantes encontraram-se fora do pr edio e concordaram no seguinte plano: o primeiro estudante entrar a na escola e abrir a todos os arm arios. O segundo aluno entrar a e fechar a todos os arm arios com n umeros pares (2, 4, 6, 8...). O terceiro aluno, ent ao, inverter a o que tiver sido feito a cada 3 arm arios (3, 6, 9...), o que signica: ele abrir a se o arm ario estiver fechado ou fechar a se o arm ario estiver aberto. O quarto aluno inverter a o que tiver sido feito a cada 4 arm arios (4, 8, 12, ...) e assim por diante. Ap os todos os alunos terem entrado e realizado suas tarefas, quais arm arios estar ao abertos? (Olimp ada Cearense/96) ABC , os tri angulos equil ateros ABM, BCN, ACP . Prove que N A = BP = CM .

Lista 8
Problema 36 (Cone Sul/92) Achar um n umero inteiro positivo n de dois algarismos de maneira que se colocarmos um 2 ` a esquerda de n, e um 1 ` a direita de n, o n umero resultante seja igual a 33n. Problema 37 Seja ABC um tri angulo is osceles com AB = AC e A = 36 . Tra camos a bissetriz de B que corta AC em D. Tra camos a bissetriz de BDC que corta BC em P . Marcamos agora um ponto R sobre a reta BC de modo que B seja o ponto m edio do segmento P R. Explique porque os segmentos RD e AP t em a mesma medida. Problema 38 Em um dado tri angulo ABC , a altura AK , a mediana CM e a bissetriz BH se encontram em um ponto O, e al em disso AO = BO. Prove que o tri angulo ABC e equil atero. Problema 39

29

poss (R ussia/2001) E vel colocarmos n umeros inteiros positivos nas casas de um tabuleiro 9 2004 de modo que a soma dos n umeros de cada linha e a soma dos n umeros de cada coluna sejam primos? Justique sua resposta. Problema 40 (Seletiva Rioplatense/2001) Em um torneio de futebol participaram os times A, B , C e D. Cada time enfrentou cada um dos outros tr es exatamente um vez. A tabela abaixo mostra o n umero de pontos ganhos, o n umero de gols pr oe o n umero de gols contra de cada equipe: Clube A B C D PG 7 4 4 1 Gp 2 5 2 2 Gc 0 2 2 7

Cada equipe recebeu 3 pontos em caso de vit oria, 1 ponto ao empatar e 0 ponto em caso de derrota. Sabendo que o jogo C D teve pelo menos um gol, determine o placar do jogo B D. Considere todos os poss veis casos.

Lista 9
Problema 41 Ache o u ltimo algarismo de N = 12 + 22 + 32 + 42 + ... + 992 . Problema 42 poss (Seletiva Rioplatense/04) E vel distribuirmos os n umeros 1, 2, 3, ..., 30 nas casas de um tabuleiro 5 6 de modo que: (a) A soma dos n umeros de cada coluna seja igual? (b) A soma dos n umeros de cada linha seja igual? Problema 43 Um trap ezio ABCD de bases BC e AD com BC < AD e tal que 2.AB = CD e BAD + CDA = 120 . Determine os a ngulos do trap ezio ABCD. Problema 44 (Rioplatense/97) H a 1997 n umeros escritos ao redor de uma circunfer encia, dos quais 1996 s ao zeros e um deles e 1. Au nica opera ca o permitida e escolher um n umero e modicar seus dois vizinhos (de 0 para 1 e de 1 para 0). (a) Demonstre que e poss vel, usando v arias vezes a opera ca o permitida, chegar a ter todos os n umeros iguais a 1. (b) Decida se com 1998 n umeros, sendo 1997 zeros e um 1, e poss vel chegar ao resultado da parte anterior. Problema 45 Fernando, Larissa e Gulherme est ao formando uma roda e se divertindo com o seguinte jogo. Um deles escolhe um n umero e o diz em voz alta, a pessoa que estiver ` a sua esquerda o divide pelo seu maior divisor primo e diz o resultado em voz alta e assim sucessivamente. Ganha a pessoa que disser em voz alta o n umero 1, e o jogo termina. Fernando escolheu um n umero maior que 50 e menor que 100 e ganhou. Na partida seguinte, Guilherme escolheu o n umero seguinte ao escolhido por Fernando e tamb em ganhou. Determinar todos os n umeros que possam ter sido escolhidos por Fernando.

30

Lista 10
Problema 46 Onofre, tamb em conhecido como Paladino, deseja cobrir um tabuleiro 7 7, e para isso disp oe de dois tipos de pe cas mostradas abaixo: (A) (B )

Onofre quer usar a menor quantidade poss vel de pe cas do tipo (B ). Quantas pe cas do tipo (B ) nosso her oi vai utilizar? D e exemplo de uma cobertura usando o m nimo poss vel de pe cas tipo (B ). Problema 47 3 DE 4 CE = e = . Sejam As diagonais AC e BD de um quadril atero convexo ABCD se cortam em E de forma que AC 7 BD 9 P e Q pontos que dividem o segmento BE em tr es partes iguais, com P entre B e Q, e R o ponto m edio do segmento AE . Calcule: area(AP QR) area(ABCD) Problema 48 (R ussia/98) Um n umero de quatro d gitos e escrito no quadro-negro. As opera co es permitidas s ao: adicionar 1 a dois d gitos vizinhos (caso nenhum deles seja 9), ou subtrair 1 de dois d gitos vizinhos (caso nenhum deles seja 0). E poss vel obtermos 2002 a partir de 1234 realizando algumas opera co es? Problema 49 (Seletiva Rioplatense/04) Em cada casa de um tabuleiro 8 8 escrevemos um n umero inteiro. Sabe-se que para cada casa, a soma dos seus vizinhos e 1. Encontre a soma de todos os n umeros do tabuleiro. Nota: Consideramos vizinhas casas com um lado em comum. Problema 50 (Rioplatense/2003) Os 5 melhores times de futebol da Am erica participaram de um torneio, onde todos jogaram contra todos uma u nica vez. Da forma usual, em caso de vit oria o time ganha 3 pontos, em caso de empate cada time ganha 1 ponto e em caso de derrota o time ganha 0 ponto. Ap os o nal do torneio foi confeccionada uma tabela especial para a Olimp ada Rioplatense, que mostra o total de pontos de cada time (Pt), o n umero de vit orias (V), a quantidade de gols a favor (Gp) e a quantidade de gols contra (Gc), onde e claro faltam alguns dados: Clube Fortaleza River Plate Santos Boca Juniors Cruzeiro PG 10 8 5 3 ? V 3 2 1 1 ? Gp 4 7 2 1 0 Gc 0 1 ? 4 6

(a) Complete a tabela acima indicando o total de pontos ganhos e o total de vit orias do Cruzeiro, e o n umero de gols contra do Santos. (b) Construa toda a tabela de jogos do campeonato indicando, com justicativa, os placares de todas as partidas.

Cap tulo 4

Solu co es
Solu c ao 1 Temos v arias solu co es para este problema. Aqui v ao duas delas (come cando em um v ertice do tri angulo e percorrendo os pontos no sentido anti-hor ario): 1, 6, 8, 2, 7, 5, 3, 4, 9 7, 3, 5, 8, 4, 2, 9, 6, 1 Obs: Seria interssante investigar mais de perto este problema e saber por exemplo quais valores pode assumir a soma S comum dos lados (no primeiro exmplo S = 17 e no segundo S = 23). Para isso veja as dicas oferecidas no problema do quadrado m agico (Problema 10). Solu c ao 2 Digamos que as idades dos lhos sejam a, b, c. Temos portanto: a + b + c = 22 Daqui a x anos, as idades dos lhos ser ao a + x, b + x e c + x enquanto a idade do pai ser a 40 + x. Queremos que: (a + x) + (b + x) + (c + x) = 40 + x (a + b + c) + 3x = 40 + x 22 + 2x = 40 x = 9

A resposta e portanto 9 anos. Solu c ao 3 Um ano n ao bissexto tem 365 dias. O dia do meio portanto seria o 183 . Basta ent ao contarmos: Janeiro : 31 dias Fevereiro: 28 dias Mar co: 31 dias Abril: 30 dias Maio : 31 dias Junho: 30 dias 31

32

At e aqui temos 181 dias. Logo o 183 dia ser a 02 de Julho. Solu c ao 4 Observe as duas u ltimas arma co es: O caj a est a` a direia da caixa vermelha, que por sua vez est a` a direita da banana. Logo devem estar dispostos assim: banana cx vermelha caj a Conclu mos portanto que o abacaxi deve estar na caixa vermelha, e que a caixa da banana e a verde, pois na primeira arma ca o ele diz que a caixa verde est a` a esquerda da azul. A congura ca o deve ser ent ao: cx verde (banana) cx vermelha (abacaxi) cx azul (caj a)

Solu c ao 5 Neste problema voc e poderia fazer uma an alise do seguinte tipo: Se Raul estiver falando a verdade, amanh a ser a realmente um dia de mentir. Nesse caso o dia da arma ca o e uma sexta-feira. Se Raul estiver mentindo, ent ao amanh a ser a dia de falar a verdade. Nesse caso a arma ca o deve ter sido feita numa ter ca-feira. Se Cida estiver falando a verdade, amanh a ser a realmente um dia de mentir, e o dia em que foi feita a frima ca o e uma ter ca. Caso Cida esteja mentindo, amanh a ser a um dia de falar a verdade, e a arma ca o deve ter sido feita num s abado. Como os dois disseram a frase no mesmo dia, conclu mos que a resposta e ter ca-feira. Solu c ao 6 Nosso her oi pode usar a seguinte estrat egia para obter os 4 litros exatos. Digamos que o balde A tenha capacidade para 5l, enquanto o balde B tem capacidade para 3l. 1. Encher o balde A 2. Derramar 3l no balde de B , sobrando 2l em A. 3. Seque ent ao o balde B e derrame os 2l restantes de A em B . Agora B cou com 2l e A cou vazio. 4. Encha novamente o balde A e em seguida complete 1 litro que falta no balde B . Sobrar ao 4 litros exatos no balde A. Obs: Voc e pode experimentar outras varia co es deste problema, como por exemplo com baldes de 7l e 5l obter 4 litros... Solu c ao 7 poss E vel sim! Imagine que ele esteja falando a frase no dia 1 de janeiro e que seu anivers ario seja no dia 31 de dezembro. Solu c ao 8 Para numerar as p aginas de 1 at e 99 ele vai usar 20 vezes o algarismo 2, dez vezes na casa das unidades: 2, 12, 22, ..., 92 e dez vezes na casa das dezenas: 20, 21, 22, ..., 29. Os pr oximos dois 2 s que v ao aparecer ser ao nas p aginas 102 e 112. Agora cuidado! Ele poder a continuar numerando as p aginas at e a 119, que e a resposta. Solu c ao 9 O enunciado diz que s o h a um cara que est a falando a verdade. Fixemos as aten co es em Eduardo e Jo ao. Para Eduardo temos duas possibilidades: ou ele mente, ou ele fala a verdade.

33

Se Eduardo mente, ent ao Jo ao estar a falando a verdade. Como s o pode haver um falando a verdade, esta pessoa s o poder a ser Eduardo ou Jo ao. De modo que temos ent ao certeza que Andr e e Rafael mentiram. Como Rafael mentiu, ele deve ser o culpado. Ent ao Eduardo deve estar mentindo tamb em, e a pessoa que falou a verdade foi Jo ao. Resposta: Rafael Solu c ao 10 O item (a) e o item (b) deste problema j a foram resolvidos no cap tulo 2. Vamos ent ao resolver o item (c). Vamos inicialmente p or letras a, b, c, ..., p para representar os n umeros 1, 2, ..., 16 em alguma ordem: a e i m b f j n c g k o d h l p

Como a soma de cada linha deve ser igual, chamemos esta soma comum de S . Montando as equa co es teremos: a+b+c+d e+f +g+h i+j+k+l m+n+o+p Somando estas quatro express oes obtemos: a + b + c + ... + p 1 + 2 + ... + 16 136 S = 4S = 4S = 4S = 34 = S = S = S = S

H a algumas maneiras diferentes de montar este tabuleiro, por em n ao vejo nenhum m etodo pr atico para tal constru ca o. Apresento a voc es tr es solu co es descobertas por alunos do Projeto Numeratizar: 4 13 16 1 6 11 10 7 9 8 5 12 15 2 3 14 10 2 13 9 1 15 8 7 12 6 5 4 11 16 3 14 1 10 7 16 14 8 9 3 15 5 12 2 4 11 6 13

Solu c ao 11 Observe que se considerarmos cada garrafa com 1 litro, haver a 10, 5 litros a serem repartidos, o que d a 3, 5 litros para cada garoto. Apresentamos aqui dois modos diferentes de se fazer a partilha: 1 modo: Garoto A: 3 cheias, 1 metade e 3 vazias. Garoto B: 3 cheias, 1 metade e 3 vazias. Garoto C: 1 cheia, 5 metades e 1 vazia. 2 modo: Garoto A: 2 cheias, 3 metades e 2 vazias.

34

Garoto B: 2 cheias, 3 metades e 2 vazias. Garoto C: 3 cheias, 1 metade e 3 vazias.

Solu c ao 12 Quando retiramos 1 litro de A para B , o barril B ca com 101 litros, numa mistura de 100 litros de alcool e 1 litro de agua. Ao realizar o segundo passo, vamos tirar 1 litro da mistura de B , digamos que venha x de alcool e (1 x) de agua. Teremos ent ao ao nal: Barril A: (100 x) agua e x alcool. Barril B : (100 x) alcool e x agua. Conclus ao: H a a mesma quantidade de alcool no barril A que de agua no barril B . Solu c ao 13 Queremos montar um n umero de 5 algarismos pal ndromo: Usaremos o princ pio fundamental da contagem. Para a primeira posi ca o (mais ` a esquerda) temos 9 possibilidades (exclui-se o ZERO). Uma vez escolhida esta posi ca o, temos 1 possibilidade para a u ltima posi ca o (deve ser um d gito igual ao primeiro). Temos 10 escolhas para o d gito da segunda posi ca o. Uma vez escolhida esta segunda posi ca o, novamente temos 1 escolha para a quarta. E por u ltimo temos 10 possibilidades para o d gito central. No total temos: 9 10 10 1 1 = 900 n umeros. Solu c ao 14 Vamos reslver este problems de tr as para a frente. (d) Os cubinhos que t em tr es faces pretas s ao exatamente os 8 cubinhos das esquinas. (c) Os cubinhos que t em exatamente duas faces pretas est ao numa aresta do cub ao inicial, mas que n ao s ao esquinas. Temos 8 cubinhos desse tipo em cada aresta, e 12 arestas, totalizando 96 destes cubinhos. (b)Os cubinhos que t em exatamente uma face preta est ao nas faces do cub ao inicial, mas n ao est ao nas arestas. Em cada face, temos 64 desse cubinhos. Como s ao 6 faces, temos 384 destes cubinhos. (a) Os cubinhos com nenhuma face preta s ao os restantes: 1000 8 96 384 = 512. Solu c ao 15 Observe que: O1 e o 1 termo. Ou ltimo 2 e o (1 + 2) termo. Ou ltimo 3 e o (1 + 2 + 3) termo. Seguindo neste racioc nio, o u ltimo n umero k e o: 1 + 2 + 3 + ... + k = k (k + 1) 2

termo. Vamos procurar k de forma que isso chegue perto de 1999. Encontramos k = 62. O u ltimo 62 ser a o 1953 termo da sequ encia. Agora vir ao 63 termos iguais a 63. Em particular o 1999 termo ser a um 63, e o resto da divis ao deste cara por 5 e 3.

35

Solu c ao 16 Faremos a contagem do seguinte modo: De 1 a 9: S ao 9 d gitos usados. De 10 a 99: S ao 90 n umeros, cada um com 2 d gitos, ou seja 90 2 = 180 d gitos utilizados. De 100 a 999: S ao 900 n umeros, cada um com 3 d gitos, ou seja 900 3 = 2700 d gitos utilizados. Como estamos procurando pelo 2004 d gito escrito, este deve aparecer em um n umero de tr es algarismos. Vamos calcular quantos d gitos escrevemos at e o n umero de tr es algarismos x. De 100 a x temos (x 100 + 1) 3 = 3x 297 algarismo escritos, e no total de 1 at e x temos: 9 + 180 + (3x 297) = 3x 108 algarismos escritos. Fazendo: vemos que o 2004 algarismo escrito e o 4 do n umero 704. Solu c ao 17 Au nica coisa que precisamos para resolver este problema e conhecer a representa ca o em base 10 dos n umeros!! Veja: IRA + ARO + ORA = = = (100I + 10R + A) + (100A + 10R + O) + (100O + 10R + A) 102A + 101O + 100I + 30R ()

3x 108 = 2004 3x = 2112 x = 704

(a) Se queremos deixar esta soma maior poss vel, devemos substituir os maiores n umeros para as letras que t em maiores coecientes na express ao (). Ou seja A = 9; O = 8; I = 3; R = 1. A maior soma ser a ent ao: S = 102.9 + 101.8 + 100.3 + 30.1 = 2056 (b) Para o c alculo da menor soma, o racioc nio e inverso. Devemos fazer A = 1; O = 3; I = 8; R = 9. A menor soma e portanto: S = 102.1 + 101.3 + 100.8 + 30.9 = 1475

Solu c ao 18 (a) Quem possui a estrat egia vencedora e o segundo jogador, ou seja ele pode sempre vencer independente de como o primeiro jogue. Sua estrat egia ser a a seguinte: ele deve jogar sempre o oposto do primeiro. Assim se o 1 jogador move a cha 1 casa para a freente, o 2 deve mov e-la 2 casas. E se o 1 move a cha 2 casas para a frente, o 2 deve mov e-la 1 casa apenas. Dessa forma o 2 jogador vai completando o tabuleiro de 3 em 3 casas e ele (2 jogador) alcan car a as casas m ultiplos de 3: 3, 6, 9, 12, 15, 18, 21, .... (b) No caso geral de um tabuleiro 1 N , se o N for m ultiplo de 3, o 2 jogador vence com a estrat egia apontada acima. Caso N n ao seja m ultiplo de 3, quem tem a estrat egia para vencer e o 1 jogador. Na sua primeira jogada ele deve deixar a quantidade de casas restantes m ultiplo de 3 (ou seja se o resto de N por 3 for 1, ele joga 1 casa, e se o resto de N por 3 for 2 ele joga 2 casas). Em seguida o 1 jogador s o deve usar a estrat egia acima de sempre completar 3 casas. Obs: Voc e pode experimentar varia co es deste jogo, onde as jogadas permitidas s ao: mover a cha 1, 2, 3, ..., ou k casas para a frente. Nesse caso a estrat egia vencedora tem a ver com a divis ao por (k + 1) e a dica e sempre completar (k + 1) casas ap os o movimento do advers ario. Solu c ao 19

36

Digamos que haja x pessoas famintas e y pessoas normais. Como 4 pizzas n ao s ao sucientes, nem que cada pessoa coma o seu m nimo de peda cos, dar a certo. Ent ao devemos ter: 6x + 2y > 48 Por outro lado, com 5 pizzas sobrariam peda cos. Isso quer dizer que mesmo que cada pessoa coma seu m aximo de peda cos, o total de 60 peda cos ser a atingido, ou seja: 7x + 3y < 60 Vamos agora testar os poss veis casos. Da segunda equa ca o obtemos que x < 9. Da : x = 8, devemos ter y = 1 que funciona!!!! x = 7, pela primeira equa ca o y 4, o que n ao funcionaria na segunda. x = 6, pela primeira equa ca o, obtemos y 7, o que n ao funcionaria na segunda os casos x = 5, 4, 3, 2, 1 tamb em n ao funcionar ao. Conclu mos portanto que 9 pessoas foram ` a pizzaria, das quais 8 estavam famintas. Solu c ao 20 Observe que ao tomarmos um tri angulo equil atero e ligarmos seus pontos m edios estamos dividindo este em 4 tri angulos equil ateros menores. Se o tri angulo equil atero original j a faz parte de uma divis ao, estaremos aumentando a quantidade de tri angulos da divis ao em 3. (a) Usando a dica acima, ao construirmos uma divis ao do tri angulo original em 1 tri angulo (ele mesmo!), conseguiremos dividi-lo em 22 triangulos menores, j a que 22 = 3.7 + 1. Veja a gura abaixo:

(b) Para construirmos uma divis ao em 17 triangulinhos, primeiro conseguimos dividir em 8 e depois acrescentamos mais 9 com a dica acima. A gura nal pode car assim:

Solu c ao 21 Desenvolvendo temos: 1094 94 = 1000...00 94 = 999..99 06


94 zeros 92 noves

37

A soma dos algarismo e portanto: (92 9) + 6 = 834

Solu c ao 22 Veja que 1722 = 2 3 7 41. As u nicas possibilidades de multiplicarmos dois n umeros de dois algarismos para obter 1722 s ao: 1722 = 42 41 = 82 21 Vamos agora ver quais s ao os verdadeiros resultados destas multiplica co es: 42 41: O verdadeiro resultado seria 24 21 = 504. 82 21: O verdadeiro resultado seria 64 41 = 2624. Como e dito no enunciado que o verdadeiro resultado seria um n umero de tr es algarismos, conclu mos que Onofre queria multiplicar os n umeros 24 e 21. Solu c ao 23 Veja a gura abaixo: Q z x A

z B

P y D y

x C

Como o ABC e is osceles, sabemos que AD tamb em e mediatriz, logo P B = P C = P Q. Veja os a ngulos marcados na gura. No BQC temos que: 2x + 2y + 2z = 180 x + y + z = 90 E da conclu mos que: P QC = x = 90 (y + z ) = 90 75 = 15

Solu c ao 24 Fatorando em primos cada n umero do conjunto {1, 2, ..., 28} e multiplicando todos esses n umeros, encontramos: P = 225 .313 .56 .74 .112 .132 .17.19.23 Para que o produto seja um quadrado perfeito todos os expoentes devem ser pares. Devemos portanto tirar os n umeros 17, 19, 23 e ainda um fator 2 e um fator 3 que pode ser feito de uma vez s o tirando o n umero 6. A reposta e portanto 4 n umeros. Solu c ao 25 Sejam a, b, c, d, e, f, g, h os oito n umeros escritos nos v ertices do cubo. Seja S a soma total dos n umeros dos v ertices, das arestas e das faces do cubo. Nas 3 arestas incidentes no v ertice a aparece uma parcela a em cada uma. Nas tr es faces incidentes no v ertice a aparece uma parcela 2a em cada uma. O mesmo vale para os outros n umeros, de modo que: S = 10(a + b + c + d + e + f + g + h)

38

Fa ca uma gura bem direitinho para vericar isto!! Veja que a soma total e m ultipla de 10, e portanto n ao pode ser igual a 2004. Solu c ao 26 Temos tr es possibilidades a analisar: Se viagem dura no m aximo 2 horas: Nesse caso apenas Evaldo teria raz ao. Se a viagem dura mais que 2 horas e no m aximo 5 horas: Nesse caso Evaldo e Raul teriam raz ao. Se a viagem dura mais que 5 horas: Nesse caso Andr ea e Raul teriam raz ao. Como e dito no enunciado que apenas um deles tem raz ao, conclu mos que a primeira das tr es possibilidades acima e a verdadeira. Solu c ao 27 Duas possibilidades s ao: 327, 654 e 981. 273, 546 e 819.

Solu c ao 28 Observando a gura do enunciado vemos que: B

A E F C (a) O lado do ABC mede 44. Veja agora que AE = 22, EF = 11, F G = 11, GD = 11, BD = 22. O per metro da area sombreada e portanto: BA + AE + EF + F G + GD + DB = 44 + 22 + 11 + 11 + 11 + 22 = 121cm 3 1 (b) A area ABDE vale da area do ABC . J aa area do EGF vale da area do 4 4 da area do ABC . Logo a raz ao entre a area sombreada e a area do ABC e: 3 1 1 3 1 13 + . = + = 4 4 4 4 16 16 EDC que por sua vez vale 1 4

Solu c ao 29 Se dois ret angulos t em mesma base, a raz ao entre suas areas e a mesma rez ao entre suas alturas. Do mesmo modo, se dois ret angulos t em mesma altura, a raz ao entre suas areas e a mesma raz ao entre suas bases. Veja gura abaixo:

39

b 2

a 1 d e X f

4 5

O ret angulo de area 3 tem mesma base que o ret angulo de area 2 (base b), assim: 3 3d e = e= d 2 2 Veja agora que o ret angulo de area 5 tem mesma base que o ret angulo de area 4 (base c), e com isso: 5 5e 15d f = f = = e 4 4 8 Lembrando que ad = 1, a area procurada vale: X = af = 15 15 ad = 8 8

Solu c ao 30 Compreendendo bem o enunciado do problema, devemos retirar um pacote para que os pacotes restantes possam ser divididos em dois grupos um com o dobro de gurinhas do outro. Digamos que o pacote retirado tenha x gurinhas e que os pacotes restantes foram divididos em dois grupos: um com y gurinhas e outro com 2y gurinhas. O total de gurinhas inicialmente e: S = 9 + 15 + 16 + 17 + 26 + 34 = 117 Quando tiramos x devemos ter: 117 x = y + 2y = 3y Assim, x deve ser m ultiplo de 3. Temos duas possibilidades: Se x = 9: Da , pela equa ca o () conclu mos que y = 36. Por em n ao h a modo de formarmos um grupo de 36 gurinhas com pacotes de 15, 16, 17, 26 e 34. Se x = 15: Pela equa ca o () descobrimos que y = 34. Agora sim! O u nico modo de formarmos um grupo com 34 gurinhas dentre as restantes e considerar o pacote de 34 como um grupo s o, e os pacotes de 9, 16, 17, 26 como o outro grupo. Conclus ao: O pacote que Z e deu ao seu irm ao tinha 15 gurinhas. Dos pacotes restantes o de 34 e de gurinhas quadradas, enquanto os pacotes de 9, 16, 17, 26 s ao de gurinhas redondas. Solu c ao 31 (a) Se tirarmos 37 bolas, pode ser que venham 9 vermelhas, 9 azuis, 9 verdes, e 10 pretas e brancas (nem todas pretas, nem todas brancas) de modo que n ao ter amos 10 bolas da mesma cor. Agora se tirarmos 38 bolas, nem que venham as 10 pretas/brancas, teremos pelo menos 28 dentre vermelhas, azuis e verdes, o que j a garante a exist encia de 10 da mesma cor. (b) Seguindo o mesmo racioc nio acima, devemos tirar 53 bolas para garantir que h a 15 de uma mesma cor. Solu c ao 32 Primeiro vamos contar os n umeros bons que come cam com 1: ()

40

Come cam por 12: 7 n umeros. Come cam por 13: 6 n umeros. Come cam por 14: 5 n umeros. Come cam por 15: 4 n umeros. Come cam por 16: 3 n umeros. Come cam por 17: 2 n umeros. Come cam por 18: 1 n umeros. De modo que come cando com 1 temos: 7+6+5+4+3+2+1= Seguindo o mesmo racioc nio temos: Come cando por 2: 6 + 5 + 4 + 3 + 2 + 1 = 21 n umeros. Come cando por 3: 5 + 4 + 3 + 2 + 1 = 15 n umeros. Come cando por 4: 4 + 3 + 2 + 1 = 10 n umeros. Come cando por 5: 3 + 2 + 1 = 6 n umeros. Come cando por 6: 2 + 1 = 3 n umeros. Come cando por 7: 1 n umero. No total temos portanto: 28 + 21 + 15 + 10 + 6 + 3 + 1 = 84 n umeros 7.8 = 28 2

Solu c ao 33 Observe a gura abaixo: A B

D Como os per metros dos tri angulos ABC e ABD s ao iguais temos: AB + BC + AC AC + BC AD + CD + AC AD + AC Somando () + () temos:

= AB + AD + BD = AD + BD () = BC + CD + BD = BC + BD ()

Agora, como os per metros dos tri angulos ACD e BCD tamb em s ao iguais:

AC + BC + AD + AC = AD + BD + BC + BD AC = BD

41

Com isso, olhando para () conclu mos tamb em que: BC = AD Por m, perceba que os tri angulos ABD e ABC s ao congruentes pelo caso LLL e com isso os a ngulos CAB e DBA s ao iguais, o que torna o tri angulo AOB is osceles e portanto AO = BO. Solu c ao 34 Aten ca o para a gurinha: P A M B 60 C

60

N O que voc e deve perceber acima e a congru encia entre os tri angulos ACN e BCP , pelo caso LAL: AC ACN CN = CP = BCP = 60 + = BC

Com isso provamos que os lados opostos ao a ngulo de (60 + ) nos dois tri angulos s ao iguais, ou seja AN = BP . De modo an alogo podemos provar que AN = CM , e com isso: AN = BP = CM

Solu c ao 35 Observe que ao nal do processo, cada arm ario d foi movimentado uma quantidade N (d) vezes, onde N (d) e o n umero de divisores positivos de d. Os arm arios que caram abertos s ao aqueles que foram operados uma quantidade mpar bem conhecidada a f de vezes, ou seja, aqueles n umeros que possuem uma quantidade mpar de divisores. E ormula k 1 2 : para calcular a quantidade de divisores positivos de um n umero fatorado em primos d = p p ...p 1 2 k N (d) = (1 + 1)(2 + 1)...(k + 1) Para que N (d) seja mpar, e necess ario portanto que cada i seja par. Isso faz de d um quadrado perfeito. Logo os arm arios buscados s ao os de n umero: 1, 4, 9, 16, 25, 36, 49, 64, 81, 100

Solu c ao 36

42

e dito no enunciado traduz-se para: Digamos que o nosso n umero seja n = ab = 10a + b. O que 2ab1 = 33 ab

2000 + 100a + 10b + 1 = 33(10a + b) 2001 = 230a + 23b = 23(10a + b) 87 = 10a + b = n A resposta e portanto n = 87. Solu c ao 37 Come camos desenhando uma boa gurinha: A
36

D
36 36 36 36

72 72

O tri angulo ABC e is osceles, logo CBA = BCA = 72 . Como BD e bissetriz temos que DBA = DBC = 36 . Isso faz com que o tri angulo ABD seja is osceles e AD = BD (marcados na gura com duas bolinhas). Al em disso, observe que os a ngulos BDP = P DC = 36 . Com isso o tri angulo BDP tamb em e is osceles e temos BP = DP . Por constru ca o RB = BP (marcamos estes segmentos com uma bolinha na gura). Observe ent ao a congru encia dos tri angulos ADP e DBR pelo caso LAL: AD = BD ADP = DBR = 144 DP = BR Com isso, os lados opostos aos a ngulos de 144 nesses tri angulos s ao iguais, ou seja AP = RD. Solu c ao 38 Observe a gura: A

H M O

B Sabemos que o ABO e is osceles, logo:

K ABO = BAO =

Como BH e bissetriz, conclu mos que OBC = . Olhando ent ao para a soma dos a ngulos no 3 = 90 = 30

ABK chegamos a:

43

Al em disso, no ABO, OM e mediana e portanto e altura tamb em, j a que este tri angulo e is osceles. Da CM A = 90 e CM passa a ser mediana e altura no tri angulo ABC , o que torna este tri angulo tamb em is osceles, com CAB = ABC = 60 . O ABC deve ser portanto equil atero. Solu c ao 39 Suponha que seja poss vel fazer tal constru ca o. Sejam L1 , L2 , ..., L9 as somas dos n umeros de cada uma das 9 linhas, e C1 , C2 , C3 , ..., C2004 as somas dos n umeros de cada uma das 2004 colunas. Como cada Li e Cj s ao primos, estes devem ser n umeros mpares (j a que s ao soma de pelo menos nove inteiros positivos). Seja S a soma de todos os n umeros do tabuleiro. Por um lado ter amos: S = L1 + L2 + ... + L9 donde conclu mos que S e mpar, pois e soma de 9 mpares. Por outro lado: S = C1 + C2 + ... + C2004 e daqui concluir amos que S e par, o que e um absurdo. Logo tal constru ca o n ao e poss vel. Solu c ao 40 Inicialmente observe que cada time jogou tr es vezes. Reproduzimos aqui a tabela do campeonato: Clube A B C D PG 7 4 4 1 Gp 2 5 2 2 Gc 0 2 2 7

Observando as pontua co es acima podemos concluir que: A obteve 2 vit orias e 1 empate. B obteve 1 vit oria, 1 empate e 1 derrota. C obteve 1 vit oria, 1 empate e 1 derrota. D obteve 1 empate e 2 derrotas. Como o time A s o fez 2 gols, suas duas vit orias devem ter sido por 1 0, e seu empate por 0 0. Temos tr es casos a considerar, dependendo contra quem foi o empate de A: 1 caso: A empata com B Nesse caso j a sabemos que: A A A 00 10 10 B C D

O outro empate do campeonato deve ter sido C D. Assim C deve ter vencido B , enquanto B vence D. Como e dito no enunciado, o empate C D e com gols. Pode ter sido 1 1 ou 2 2 j a que C s o faz dois gols. Por em, se tivesse sido 2 2, C j a levaria 3 gols (pois levou 1 de A) o que n ao pode. Logo, chegamos a mais um resultado: C 11 D

44

Com isso sobra 1 gol apenas para C fazer em sua vit oria, donde conclu mos que: B 01 C E contando os gols que faltam, chegamos a: B 51 D 2 caso: A empata com C Nessa op ca o sabemos que: A A A 10 00 10 B C D

Nesse caso B D deve ser empate, como B precisa de 1 vit oria, B deve ter vencido C , e com isso C deve ter vencido D. Agora observe que B deve marcar 5 gols. Para vencer C , B n ao pode marcar mais de 2 gols, pois C s o leva 2 gols. Assim B deve fazer pelo menos 3 gols em D, mas a esse jogo n ao poderia ser empate j a que D s o marca 2 gols. Este caso n ao tem solu ca o. 3 caso: A empata com D Agora teremos a seguinte situa ca o: A A A 10 10 00 B C D

O outro empate deve ser ent ao entre B e C . Como D j a empatou com A, D deve perder para B e C . Como D n ao leva gol de A, deve levar seus 7 gols de B e C . Por em B e C marcam juntos exatamente 7 gols, que devem ent ao ser disparados todos em D. Achamos portanto o resultado do empate B C : B 00 C Como B deve levar 2 gols, ser a 1 do A e 1 do D, e portanto temos os placares que faltam: B 51 D C 21 D Curiosamente, a resposta e a mesma para os dois poss veis casos: B 5 1 D. Solu c ao 41 Denotaremos por ..x um n umero que acaba no algarismo x. Veja que a sequ encia dos u ltimos algarismos se repete de dez em dez: N N N (..1 + ..4 + ..9 + ..6 + ..5 + ..6 + ..9 + ..4 + ..1 + ..0) + (..1 + ..4 + ... 10(..1 + ..4 + ..9 + ..6 + ..5 + ..6 + ..9 + ..4 + ..1 + ..0) 10.(..5) 0

Vemos portanto que N acaba em ZERO. Solu c ao Alternativa: Usando a f ormula para o c alculo da soma dos quadrados temos: N= 99.100.199 33.50.199 0 (mod 10) 6

45

Solu c ao 42 Veja que a soma de todos os n umeros do tabuleiro e: S = 1 + 2 + ... + 30 = 30.31 = 15.31 = 465 2

(a) Como 465 n ao e divis vel por 6 (quantidade de colunas) n ao e poss vel distribuir os n umeros no tabuleiro para que a soma de cada coluna seja igual. (b) Em contrapartida, como 465 = 93 5, j a temos um ind cio de que e poss vel fazer a distribui ca o para deixar a soma de cada uma das 5 linhas igual (a 93). O exemplo abaixo mostra que de fato isto e poss vel: 1 4 7 10 13 30 27 24 21 18 2 5 8 11 14 29 26 23 20 17 3 6 9 12 15 28 25 22 19 16

Solu c ao 43 Observe a gura abaixo: B C


60

x M x

x A

Por C tra camos uma paralela ao lado AB encontrando AD em J . Veja que: CJD + CDJ = BAD + CDA = 120 JCD = 60 Seja M o ponto m edio de CD. O JCM e is osceles, pois JC = CM = x, e tem a ngulo do v ertice C = 60 . Com isso o JCM deve ser equil atero e teremos JM = x. Observe que: JM = CM = DM = x Conclu mos portanto que o CJD deve ser ret angulo em J , o que faz com que = 90 e = 30 . Os outros a ngulos dos trap ezio s ao BCD = 150 e CBA = 90 . Solu c ao 44 (a) Considere um bloco de 4 zeros consecutivos na circunfer encia: 0000 Se realizarmos a opera ca o no segundo zero do bloco, modicaremos seus dois vizinhos e caremos com: 1010 Agora se realizarmos a opera ca o na terceira posi ca o (o 1) modicaremos seus dois vizinhos para car com: 1111 Conseguimos ent ao com estes dois passos trocar quatro zeros consecutivos por quatro 1 s. Basta ent ao dividir os 1996 zeros em 499 blocos de 4 zeros consecutivos e aplicar o esquema acima. (b) Com 1997 zeros e um 1 inicialmente, n ao e poss vel transformar tudo em 1. Para provar isso considere a soma S dos n umeros da circunfer encia. Vejamos o que pode acontecer com S quando realizamos uma opera ca o:

46

Se a casa operada tem dois vizinhos 0: Nesse caso S se transformar a em S + 2. Se a casa operada tem um vizinho 0 e outro vizinho 1: A soma S permanecer a constante. Se a casa operada tiver dois vizinhos 1: A soma S diminuir a para S 2. Veja que quando realizamos a opera ca o, a paridade de S n ao muda. Inicialmente S = 1. Logo, n ao podemos ter ao nal todos os n umeros iguais a 1, pois nesse caso ter amos S = 1998, que seria uma contradi ca o. Solu c ao 45 Observe que o jogador que diz o n umero inicial N ganha o jogo quando a soma dos expoentes da fatora ca o em primos de N e m ultipla de 3 (pois a cada rodada um fator primo sai, e como se um expoente baixasse 1). A maneira mais na ra ca de se fazer este problema e ent ao e fatorar todos os n umeros entre 50 e 100 e ver os pares de n umeros consecutivos cujas somas dos expoentes sejam m ultiplas de 3. Ap os esta an alise de casos, encontramos as respostas (o primeiro n umero e o que foi dito por Fernando): 63 = 32 .7 75 = 3.52 98 = 2.72 e e e 64 = 26 76 = 22 .19 99 = 32 .11

Solu c ao 46 Digamos que Onofre v a utilizar x pe cas do tipo (A) e y pe cas do tipo (B) para cobrir o tabuleiro. Cada pe ca do tipo (A) tem 4 casas enquanto cada pe ca do tipo (B) tem 3 casas. O tabuleiro 7 7 tem 49 casas, o que nos permite montar a seguinte equa ca o: 4x + 3y = 49 Como y deve ser o menor poss vel, testemos estes valores: y = 0 4x = 49, y = 1 4x = 46, y = 2 4x = 43, Absurdo! Absurdo! Absurdo!

y = 3 4x = 40 x = 10 Pelo que foi visto acima, n ao podemos cobrir o tabuleiro usando apenas 0, 1 ou 2 pe cas (B). O menor valor poss vel seria 3, como mostra o exemplo abaixo:

Solu c ao 47 Para resolver este problema usaremos o seguinte resultado: se dois tri angulos t em mesma altura, a raz ao entre suas areas e a mesma raz ao entre suas bases. Em particular quando um tri angulo e dividido em dois por meio de uma ceviana, a raz ao entre suas areas e a mesma entre suas bases. Veja a gura abaixo:

47

C D E Q R P B 4 DE 4 DE = , temos que = . Assim: BD 9 BE 5

A Vamos denotar areas por par enteses. Chamemos (AEB ) = x. Como DE 4 (ADE ) = = BE 5 (AEB ) donde conclu mos que (ADE ) = 4x . Pelo mesmo racioc nio, temos: 5

(CEB ) CE 3 3x = = (CEB ) = (AEB ) AE 4 4 Ainda de modo an alogo: DE 4 4 3x 3x (CED) = = (CED) = . = (CEB ) BE 5 5 4 5 Logo a area do quadril atero ABCD ser a: (ABCD) = = = (ABE ) + (ADE ) + (CEB ) + (CED) x+ 63x 20 EQA e 4x 3x 3x + + 5 4 5

Uma mediana divide um tri angulo em duas partes de mesma area. Sabendo disso, como QR e mediana no AP e mediana no QAB temos: (ERQ) = (RQA) e (QAP ) = (P AB ) e isso claramente implica em: (AP RQ) = A resposta para o nosso problema e portanto: (AP QR) x 20 10 = . = (ABCD) 2 63x 63 1 x (ABE ) = 2 2

Solu c ao 48 O que ser a que n ao muda em um n umero quando adicionamos ou subtra mos 1 unidade de dois d gitos vizinhos? Por meio desta opera ca o veja que podemos estar somando ou subtraindo: 11, 110 ou 1100 do n umero original. E o que estas parcelas t em todas em comum? S ao m ultiplas de 11! Ou seja estamos sempre adicionando ou subtraindo um n umero m ultiplo de 11 do n umero corrente, de modo que o resto da divis ao por 11 do n umero escrito no quadro n ao muda. Como come camos com 1234 que deixa resto 2 por 11, n ao podemos chegar a 2002 que deixa resto 0 por 11.

48

Solu c ao 49 Observe as casas marcadas no tabuleiro abaixo: * * * * * * * * * * * * * * * * * * * *

Se olharmos para os vizinhos das casas marcadas acima, vemos que eles cobrem todo o tabuleiro e de maneira disjunta! Como a soma dos vizinhos de cada casa e 1, a soma total dos n umeros do tabuleiro ser a igual ao n umero de casas marcadas, que e 20. Solu c ao 50 Reveja a tabela do campeonato: Clube Fortaleza River Plate Santos Boca Juniors Cruzeiro PG 10 8 5 3 ? V 3 2 1 1 ? Gp 4 7 2 1 0 Gc 0 1 ? 4 6

(a) Inicialmente veja que o total de gols marcados por todos os times deve ser igual ao total de gols sofridos por todos os times: Gp = 4 + 7 + 2 + 1 + 0 = 0 + 1+? + 4 + 6 = Gc Conclu mos portanto que o Santos sofreu 3 gols. O campeonato teve ao todo 10 jogos, como e indicado na tabela, 7 deles j a acabaram com vit orias (3 + 2 + 1 + 1). Veja ainda que Fortaleza teve 1 empate, River teve 2 empates e Santos teve 2 empates, o que somados j a fornecem 5 empates, ou seja pelo menos 3 jogos acabaram empatados. Esses s ao os 3 jogos restantes. Logo o Cruzeiro teve 0 vit orias. Vejamos agora o total de pontos do campeonato, houve 7 jogos com vit oria (3 pontos cada) e 3 jogos empatados (2 pontos cada) o que d a um total de: (7 3) + (3 2) = 27 pontos Portanto o Cruzeiro deve ter feito 1 ponto. (b) Vamos agora montar a tabela de jogos do campeonato. Fortaleza teve 3 vit orias e 1 empate. Esse empate deve ter sido com o River que tamb em n ao perdeu (2 vit orias e 2 empates). Como o Fortaleza n ao sofreu gols, tiramos: F ortaleza 0 x 0 River Logo o Fortaleza venceu Santos, Boca e Cruzeiro. O Boca obteve apenas uma vit oria que deve ter sido sobre o Cruzeiro (pois o jogo Boca x Cruzeiro n ao admite outro resultado). Sendo assim River e Santos vencem o Boca. Para o Santos restam 2 empates que s ao com River e Cruzeiro. O u ltimo jogo que e River e Cruzeiro acaba com vit oria do River. Vamos agora aos placares. O empate Cruzeiro x Santos foi sem gols, pois o Cruzeiro n ao marcou gols no campeonato: Cruzeiro 0 x 0 Santos Como o Boca marca s o um gol no campeonato, este deve ser para consolidar sua vit oria, da : Boca 1 x 0 Cruzeiro

49

O Fortaleza tem 4 gols para fazer 3 vit orias, logo duas delas ser ao 1 x 0 e outra ser a 2 x 0. Se a vit oria contra o Santos fosse de 1 x 0, ao Santos sobrariam 2 gols pro e 2 gols contra para uma vit oria e um empate o que n ao e poss vel. Logo: F ortaleza 2 x 0 Santos E ainda: F ortaleza 1 x 0 Boca F ortaleza 1 x 0 Cruzeiro Resta ao Santos sofrer 1 gol, que deve ser para o River, j a que o Boca n ao marca mais. Como o jogo Santos x River termina empatado devemos ter: Santos 1 x 1 River A vit oria do Santos ser a ent ao de: Santos 1 x 0 Boca Resta ao Boca levar 2 gols do River, e n ao fazer nenehum, logo: River 2 x 0 Boca Por m, o River deu uma goleada no Cruzeiro, para completar seus gols pro: River 4 x 0 Cruzeiro

50

Ap endice A

Sugest oes de Conte udo


Aqui v ao ent ao algumas sugest oes para que voc e monte seu programa de ensino:

4a s erie
Carga hor aria proposta: 3h/aula por semana - curso de 40 semanas 1. Teoria dos conjuntos - 9h 2. N umeros inteiros (opera co es); N umeros fracion arios; Express oes Num ericas - 12h 3. Divisores de um n umero; N umeros primos; Fatora ca o em primos; M etodo para encontrar divisores; Regras de divisibilidade; MDC; Algoritmo de Euclides; MMC - 24h 4. Fra co es II (mais problemas); Bases de Numera ca o; D zimas Peri odicas; Propor co es, Regra de tr es - 24h 5. Equa co es, manipula co es alg ebricas com uma vari avel; Sistemas de equa co es - 12h 6. Geometria plana: Ponto, reta, segmentos de reta; Sistema m etrico decimal; Areas de guras planas e problemas; Volumes de s olidos, problemas - 21h 7. Aulas de problemas; Simulados - 18h

5a s erie
Carga hor aria proposta: 4h/aula por semana - curso de 40 semanas Aritm etica: 1. Teoria dos Conjuntos - 8h 2. N umeros inteiros e fracion arios, express oes num ericas; Bases de Numera ca o e d zimas peri odicas - 16h 3. Propor co es e Regra de tr es - 10h 4. Divisores de um n umero, N umeros primos, Fatora ca o em primos, M etodo para encontrar divisores, Regras de divisibilidade, MDC, Algoritmo de Euclides, MMC - 22h 5. Equa co es com 1 vari avel e Sistemas de equa co es - 12h Geometria: 1. Ponto, reta, segmento de reta, a ngulos; Sistema m etrico decimal; Areas de guras planas e problemas; Volumes de s olidos, problemas - 28h

51

52

2. Paralelismo e a ngulos - 12h 3. Congru encia de tri angulos - 12h 4. Teorema de Pit agoras - 4h Combinat oria: 1. Introdu ca o aos princ pios da contagem - 12h 2. Princ pio da casa e dos pombos, L ogica Matem atica; Paridade, Jogos e Invariantes - 24h

6a s erie
Carga hor aria proposta: 4h/aula por semana - curso de 40 semanas Aritm etica 1. Teoria dos Conjuntos - 6h 2. N umeros inteiros e fracion arios, express oes num ericas; Bases de Numera ca o e d zimas peri odicas - 12h 3. Propor co es e Regra de tr es - 6h 4. Equa co es com 1 vari avel e Sistemas de equa co es - 6h 5. Divisores de um n umero, N umeros primos, Fatora ca o em primos, M etodo para encontrar divisores, Regras de divisibilidade, MDC, Algoritmo de Euclides, MMC - 16h 6. Algoritmo da Divis ao - 8h Geometria(Bibliograa: Matem atica Elementar, vol 9) 1. Introdu ca o, segmento de reta; Angulos; Revis ao: Sistema M etrico, Areas, Volumes - 12h 2. Tri angulos, Congru encia de tri angulos, Desigualdades no tri angulo - 12h 3. Paralelismo; Perpendicularidade - 12h 4. Quadril ateros Not aveis (8h); Circunfer encia, area e per metro (4h); Pontos Not aveis no tri angulo (8h) - 20h 5. Semelhan ca de tri angulos e Teorema de Pit agoras - 10h Combinat oria-Algebra 1. Introdu ca o aos problemas de Olimp ada - 6h 2. Princ pios da Contagem, Permuta co es, Combina co es, Permuta co es Circulares - 14h 3. No co es de Algebra: Produtos Not aveis, Ra zes e Pot encias, M odulos, Racionaliza ca o de denominadores - 10h 4. Princ pio da casa e dos pombos; Paridade; Jogos e Invariantes - 10h

7a s erie
Carga hor aria proposta: 4h/aula por semana - curso de 40 semanas Algebra 1. Introdu ca o ` a L ogica Matem atica - 4h

53

2. Produtos Not aveis e fatora co es - 8h 3. Radicia ca o, Potencia ca o, Fun ca o Modular; Revis ao: Equa co es do 1o grau, Sistemas de equa co es, Propor co es, Regras de tr es, muitos e muitos problemas - 10h 4. Revis ao: Bases de Numera ca o, D zimas peri odicas - 6h 5. Equa co es do 2o grau (tudo) e fun ca o quadr atica - 10h 6. Progress oes Aritm eticas e Geom etricas - 6h 7. Desigualdades entre as m edias - (se der tempo) Geometria (Bibliograa: Matem atica Elementar, vol.9) 1. Revis ao: Introdu ca o, retas, segmentos de reta, a ngulos; Revis ao: Sistema m etrico decimal, per metros, areas e volumes - 8h 2. Tri angulos: Congru encias e Desigualdades (8h); Paralelismo e Perpendicularidade (6h); Quadril ateros Not aveis (4h); Pontos not aveis no tri angulo (6h) - 24h 3. Pol gonos: Diagonais e a ngulos (2h); Circunfer encia: Posi co es, a ngulos, tangentes, quadril ateros inscrit veis (8h) - 10h 4. Teorema de Tales e Teorema da bissetriz (4h); Semelhan ca de tri angulos, pot encia de ponto (6h); Tri angulo ret angulo: Rela co es M etricas e Teorema de Pit agoras (6h) - 16h Aritm etica - Combinat oria 1. Revis ao: N umeros inteiros, fracion arios, radicais, pot encias, m odulos, d zimas peri odicas - 4h 2. Divisores de um n umero, N umeros primos, Fatora ca o em primos, M etodo para encontrar divisores, Regras de divisibilidade, MDC, Algoritmo de Euclides, MMC - 10h 3. Algoritmo da Divis ao e Congru encias (introdut orio) (12h); Equa co es diofantinas (4h) - 16h 4. Princ pios da Contagem, Permuta co es, Combina co es e Permuta co es Circulares - 12h 5. Princ pio da Indu ca o - 6h 6. Paridade, Jogos e Invariantes, Princ pio da Casa e dos Pombos - 10h

8a s erie
Carga hor aria proposta: 6h/aula por semana - curso de 40 semanas Geometria Bibliograa: Matem atica Elementar, vol 9; Challenging Problems in Geometry, I.M. Yaglom Geometry Revisited, H.S.Coxeter Programa: 1. Tri angulos: Congru encias e Desigualdades; Paralelismo e perpendicularidade - 16h 2. Quadril ateros Not aveis (4h); Pontos not aveis no tri angulo (propriedades b asicas); Teorema de Tales, Teorema da bissetriz, semelhan ca de tri angulos (8h); Tri angulos Ret angulos e o teorema de Pit agoras (4h) - 16h

54

3. Circunfer encia: a ngulos, tangentes, pot encia de ponto; Quadril ateros inscrit veis e o teorema de Ptolomeu - 14h 4. Teoremas de Ceva e Menelaus - 6h 5. Rela co es m etricas no tri angulo qualquer; Areas; Lei dos senos e Trigonometria - 14h 6. Transforma co es Geom etricas: Homotetia, Rota ca o, Reex ao, Transla ca o - (opcional); O tri angulo e seus c rculos: tri angulo ortico, tri angulo medial, o inc rculo e exinc rculos, tri angulo pedal, reta de Simpson, a reta de Euler e o c rculo dos nove pontos - (opcional) Algebra 1. Produtos not aveis e fatora co es - 8h 2. Potencia ca o, Radicia ca o, Racionaliza ca o, Radicais Duplos, Fun ca o Modular - 4h 3. Revis ao de 1o grau: Propor co es, Regra de tr es, bases de numera ca o, d zimas peri odicas, sistemas de equa co es, muitos e muitos problemas - 8h 4. Equa ca o do 2o grau e fun ca o quadr atica - 10h 5. Inequa co es(4h); Progress oes (4h); Desigualdades entre as m edias (10h); Fun co es e Polin omios (8h) - 26h 6. Resolu ca o de Exerc cios - aulas restantes Teoria dos N umeros 1. Divisores, n umeros primos, fatora ca o em primos, regras de divisibilidade, MDC, Algoritmo de Euclides, MMC 10h 2. Princ pio da Indu ca o - 6h 3. Algoritmo da divis ao e congru encias (6h); MDC e MMC (aprofundamento)(4h); Equa co es diofantinas (4h) - 14h 4. N umeros primos (6h); Fun ca o parte inteira (4h) - 10h 5. Congru encias em Z (parte II) (6h); Ternos pitag oricos; Equa co es Diofantinas II (6h) - 12h 6. Teoermas de Fermat, Euler e Wilson - 6h 7. Teorema Chin es dos Restos (4h); A fun ca o phi de Euler (4h) - 8h 8. Resolu ca o de exerc cios - aulas restantes Combinat oria 1. Princ pios da Contagem; Permuta co es e Permuta co es Circulares; Combina co es; Combina co es completas; Probabilidade - 16h aulas 2. Bin omio de Newton, tri angulo de Pascal e rela ca o de Stifel (opcional) 3. O princ pio da casa e dos pombos - 6h 4. O princ pio da Inclus ao - Exclus ao (opcional) 5. Paridade; Jogos e Invariantes (10h); Teoria dos Grafos (8h); Seq u encias recorrentes (8h) - 26h 6. Resolu ca o de Problemas - aulas restantes

1o/2o anos
Carga hor aria proposta: 6h/aula por semana - curso de 40 semanas Geometria Bibliograa:

55

Matem atica Elementar, vol 9; Challenging Problems in Geometry, I.M. Yaglom Geometry Revisited, H.S.Coxeter Programa: 1. Paralelismo e Perpendicularismo. Congru encia de tri angulos e desigualdade triangular - 8h 2. Semelhan ca de tri angulos, teorema de Tales, teorema da bissetriz - 6h 3. Pontos Not aveis no tri angulo (I) (4h); Rela co es m etricas no tri angulo ret angulo. Teorema de Pit agoras. Rela co es m etricas no tri angulo qualquer, lei dos senos, lei dos cossenos, rela ca o de Stewart, f ormulas de area (8h) - 12h 4. Quadril ateros Inscrit veis e o teorema de Ptolomeu; Teoremas de Ceva e Menelaus - 8h 5. Os c rculos do tri angulo: inc rculo, exinc rculos e circunc rculo (4h); Pontos Not aveis (II): o tri angulo ortico, o tri angulo medial, a reta de Euler e o c rculo dos nove pontos, tri angulo pedal, reta de Simpson, sim etricos do ortocentro e outras propriedades; Teorema de Napole ao (10h) - 18h 6. Geometria com Trigonometria - 8h 7. Transforma co es Geom etricas: Homotetia, Rota ca o, Reex ao, Transla ca o - 8h 8. Geometria Anal tica para Olimp ada (opcional) Algebra 1. Produtos Not aveis e Fatora co es - 6h 2. Equa ca o do 2o grau e Fun ca o quadr atica - 6h 3. Inequa co es; Sequ encias e Progress oes - 6h 4. Polin omios, rela co es de Girard (6h); N umeros Complexos e ra zes da unidade (6h) - 12h 5. Matrizes, determinantes e sistema lineares - 4h 6. Desigualdades (I) : M edias, Cauchy-Schwarz (8h); Desigualdades (II): Rearranjo, Chebychev e outras elementares (opcional)- 8h 7. Equa co es Funcionais - 12h 8. Resolu ca o de problemas - aulas restantes Teoria dos N umeros 1. Princ pio da Indu ca o - 6h 2. Algoritmo da Divis ao e Congru encias (8h); MDC, MMC e Divisibilidade (6h) - 14h 3. Equa co es diofantinas (I) - 6h 4. N umeros primos; Fun ca o parte inteira - 6h 5. Ternos Pitag oricos; Teoremas de Euler-Fermat, Wilson e do Resto Chin es; A fun ca o phi de Euler - 12h 6. Equa co es Diofantinas (II) - 8h 7. Resolu ca o de Problemas- aulas restantes Combinat oria 1. Contagem: princ pio fundamental, permuta co es, combina co es, permuta co es circulares e combina co es completas; Probabilidade - 12h

56

2. Princ pio da Inclus ao- Exclus ao; N umeros binomiais, tri angulo de Pascal e Bin omio de Newton - 8h 3. Princ pio da Casa e dos Pombos - 6h 4. Contagem dupla (8h); Sequ encias recorrentes (8h) - 16h 5. Teoria dos Grafos (I) (10h); Jogos e Invariantes (14h) - 24h 6. Aulas restantes: Exerc cios

You might also like